Wikipedia:Reference desk/Science

From Wikipedia, the free encyclopedia

This is an old revision of this page, as edited by Tango (talk | contribs) at 11:26, 9 April 2013 (→‎Orbital Debris). The present address (URL) is a permanent link to this revision, which may differ significantly from the current revision.

Welcome to the science section
of the Wikipedia reference desk.
Select a section:
Want a faster answer?

Main page: Help searching Wikipedia

   

How can I get my question answered?

  • Select the section of the desk that best fits the general topic of your question (see the navigation column to the right).
  • Post your question to only one section, providing a short header that gives the topic of your question.
  • Type '~~~~' (that is, four tilde characters) at the end – this signs and dates your contribution so we know who wrote what and when.
  • Don't post personal contact information – it will be removed. Any answers will be provided here.
  • Please be as specific as possible, and include all relevant context – the usefulness of answers may depend on the context.
  • Note:
    • We don't answer (and may remove) questions that require medical diagnosis or legal advice.
    • We don't answer requests for opinions, predictions or debate.
    • We don't do your homework for you, though we'll help you past the stuck point.
    • We don't conduct original research or provide a free source of ideas, but we'll help you find information you need.



How do I answer a question?

Main page: Wikipedia:Reference desk/Guidelines

  • The best answers address the question directly, and back up facts with wikilinks and links to sources. Do not edit others' comments and do not give any medical or legal advice.
See also:


April 5

Stars that becomes a white dwarf

Since our article said Although white dwarfs are known with estimated masses as low as 0.17 and as high as 1.33 solar masses, the mass distribution is strongly peaked at 0.6 solar mass, and the majority lie between 0.5 to 0.7 solar mass do we know exactly how much our sun will lose mass by the time it reaches white dwarf. I scientist already knew exactly how much mass will sun lost by the time it gets to white dwarf. I didn't think the 3 solar mass stars will become white dwarf, I remember in my astronomy class I took, it was star between 0.7 to 1.4 solar mass stars becomes white dwarf. But White dwarf article said 8 solar mass stars become white dwarf.--69.226.42.134 (talk) 00:12, 5 April 2013 (UTC)[reply]

When a star becomes a white dwarf doesn't just depend on how massive it is, it also depends on what it is composed of and how massive it is once it has fused most of its hydrogen and helium. The white dwarf article explains that a star of 8 solar masses is massive enough to fuse oxygen and helium into carbon, but not massive enough to fuse carbon. So once that process is finished, that's when it becomes a white dwarf. As for the fate of our sun, see Sun#After_core_hydrogen_exhaustion. 203.110.235.3 (talk) 05:06, 5 April 2013 (UTC)[reply]
See also Formation_and_evolution_of_the_Solar_System#The_Sun_and_planetary_environments. Ruslik_Zero 19:29, 5 April 2013 (UTC)[reply]

Hydrogen spectrum picture in hydrogen article

Is this picture showing four different spectral frequencies that hydrogen emits on? (It is only visible in the enlarged picture.)My understanding is that there would be two places in the spectrum to find hydrogen. A positive and negatively charged frequency. If that is wrong, or if I am misinterpreting what the vertical lines indicate a clarification would help me greatly. I am working on isolating and transmitting hydrogen from the spectrum.64.91.201.2 (talk) 01:02, 5 April 2013 (UTC)[reply]

There are 5 lines, not 4, all of which are from the Balmer series. "Positive and negative charged frequency" is meaningless. "Isolating and transmitting hydrogen from the spectrum" has no discernible meaning. --140.180.248.141 (talk) 03:15, 5 April 2013 (UTC)[reply]
The Balmer series is those hydrogen lines that appear in the visible spectrum, but the entire spectrum of hydrogen can be derived from first principles by the Rydberg formula; that formula was an important keystone in building modern atomic theory as it provided the mathematical underpinning of the Bohr model; it is vital to note that Rydberg arrived at his formula quite independently of Bohr, or indeed any quantum theory at all, as the formula predates the Bohr model by 25 years. Rydberg's formula was merely a descriptor of the lines in the hydrogen spectrum, but Bohr used it as the key piece of evidence in his quantum absorption-transmission model of electron behavior. The "n" values in Rydberg's formula were later shown to be functionally identical to the principal quantum number as understood by modern quantum theory. --Jayron32 04:40, 5 April 2013 (UTC)[reply]

SPACE TRAUMA

Please check whether fact no. 6 of this linked article under the heading "Space Trauma" is right or wrong. Britannica User (talk) 06:12, 5 April 2013 (UTC)[reply]

It's Right. People will not explode in space.
See Space exposure
and Effect_of_spaceflight_on_the_human_body#Direct_exposure_to_the_extreme_environment_of_space
Basically stepping outside a spaceship would cause a lot of problems for you, but the first thing to become fatal would be simple suffocation. You'd lose consciousness after about 10 seconds and be completely dead after about 90 seconds. (Why so much faster than drowning? It's impossible to hold your breath in space. The pressure difference is too great.) APL (talk) 06:23, 5 April 2013 (UTC)[reply]
yeah. if your skin would explode from 1 atmosphere pressure differential, you could presumably do major local damage to yourself with the nozzle of a good vacuum cleaner. doesn't happen. For something like liposuction, for instance, it requires a major amount of physical digging to loosen the tissue, the vacuum itself won't rip it loose. Gzuckier (talk) 20:13, 5 April 2013 (UTC)[reply]
Also of interest; Rapid_decompression#Exposure_to_a_vacuum_causes_the_body_to_explode. 203.27.72.5 (talk) 05:50, 8 April 2013 (UTC)[reply]

Important Questions (about brains and magnets)

1. It is a common phenomenon that a bar magnet, when suspended from a string, align along the south to north direction. Is there any lurid description of this phenomenon? Why don't it align in east to west direction?
2. Why do two magnets attract(or repel) each other?
3. Is it true that every multicellular organism has a brain? If not, then how they function. And is there any multicellular organism having more than one brain? I have heard that a snail has 32 brains; is it true? Scientist456 (talk) 07:21, 5 April 2013 (UTC)[reply]

I added to your title to make it useful. StuRat (talk) 07:46, 5 April 2013 (UTC) [reply]
1) I'm guessing you want a lucid description, not a lurid one, although a lurid one might be more fun: "The magnetic line of force approaches the magnet silently from behind, and flings it recklessly about, until the final act, when it pins it mercilessly in one spot, never relenting.".
2) Physics isn't very good at explaining why basic forces of nature behave as they do, it simply observes that they do, and comes up with formulas to describe how they behave. The basic observation for magnets is that opposites attract and likes repel (so only the middle pair in the drawing below attract). We can put that into a formula, but that still doesn't explain why.
+--------+   +--------+   +--------+   +--------+
| +    - | R | -    + | A | -    + | R | +    - | 
+--------+   +--------+   +--------+   +--------+
3) This will depend on how you define "brain". The most basic definition might be a nerve cluster, in which case humans have multiple brains, too. StuRat (talk) 07:54, 5 April 2013 (UTC)[reply]
Mesozoans, Placozoans and Sponges are multicellular animals that have no nervous system at all. --Guy Macon (talk) 08:24, 5 April 2013 (UTC)[reply]
The most common (extremely common too!) multicellular organism without a brain is the entire plant family. Wickwack 121.221.88.3 (talk) 10:38, 5 April 2013 (UTC)[reply]
Not the entire plant family (more accurately plant kingdom). See green algae. Although I do believe that all embryophytes (commonly called"land plants") are multicellular. Equisetum (talk | contributions) 12:46, 5 April 2013 (UTC)[reply]
there is a school of thought that considers a bunch of "unicellular" organisms floating around to be a multicellular organism, just one where the cells aren't affixed to each other. even when the cells involved are of multiple species, genera, kingdoms, etc. Gzuckier (talk) 20:18, 5 April 2013 (UTC)[reply]
1) The reason that a magnet (and a magnetic compass) aligns North-South instead of East-West is that the Earth itself behaves like a bar magnet, aligned approximately through the North and South Poles. The cause is ultimately the rotation of the Earth, through currents in the outer core, probably caused by interaction between lodestone (or more marginally magnetised magnetite or pyrrhotite), convection currents and the Coriolis effect (there are various theories). What we call the north pole of a magnet is more correctly called the "north-seeking" pole (because it is attracted by the "south-seeking" pole of the Earth near to the North Pole. (Yes, I know that's confusing, but just remember that the pole of a magnet that points north is called the north pole, whilst the North pole is a geographical concept.) You might be interested in the article on Earth's magnetic field. Dbfirs 11:39, 5 April 2013 (UTC)[reply]
Trees are multicellular, and have no brain. To expand on StuRat's decent answer, a cluster of nerves cell bodies is called a ganglion; your brain is basically a giant, specialized ganglion (more properly, the core of the brain is the basal ganglion, where as the brain is sort of organized around that). Most animals have brains of some sort, but there are some animals where the "brain" (that is, the ganglion in the head) is not significantly larger than other ganglia in other parts of the animal, in these cases you sometimes here the idea that the animal has multiple "brains", but properly the brain is the ganglia in the head. As I noted in the first sentence, plants have no nerves and no brains, but are multicellular. In the animal kingdom, Cnidaria (primitive sea creatures including jellyfish, sea anemones and coral) have no brains at all, neither do other related phylla of sea creatures like Ctenophora and the Sponges.--Jayron32 12:23, 5 April 2013 (UTC)[reply]
2) I am not sure if anyone knows 'why' things attract or repel each other (magnets, charges, bodies etc). I guess it has something to do with Fundamental_forces - WikiCheng | Talk 12:27, 5 April 2013 (UTC)[reply]
It depends on the meaning of "why". "Why" can mean "by what mechanism does it work". We know that answer. Magnets attract because of the attractive nature of the electromagnetic force, and is best modeled as the behavior of a magnet within a Magnetic field. Field theory is not simple to understand, but it is a powerful means of describing the operation of forces. If you're using the word "why" to mean "for what purpose do magnets exist" (the same usage as "Why did you do that?"), then we can't answer that question except to note that magnets work because they obey the rules that govern the rest of the universe, and that those rules exist in concert; you can't change any of them without fundamentally altering how the universe works, and if you did that, then we wouldn't exist to study how magnets work in the first place (this is called the weak anthropic principle and is, philosophically, the best answer we have to "Why does <insert law of physics here> work the way it does and not some other way.") --Jayron32 12:37, 5 April 2013 (UTC)[reply]
All physical systems "prefer" to be in a state of lower energy. If they're not at a local minimum of the energy, there is a force on the system in the direction of decreasing energy. Magnetic fields have energy. If you superimpose two bar magnets oriented the same way, you get twice the field strength of one bar magnet alone, which gives you four times the energy (it goes like the field strength squared). If you superimpose them pointing in opposite directions, the fields cancel out and the energy is zero (or at least much smaller). If you separate the magnets by a large distance, they don't interact and the total energy is twice the energy of one alone. Therefore, magnets prefer being together with opposite poles touching to being far apart, and prefer being far apart to being together with like poles touching. -- BenRG 06:20, 7 April 2013 (UTC)


Now, I want to know whether a snail(or any species of snail) has 32 brains or not? 27.62.111.6 (talk) —Preceding undated comment added 12:39, 5 April 2013 (UTC)[reply]

It doesn't. It has six ganglia (see Jayron's answer above). The six ganglia, which you could have quickly discovered youself by googling "snail anatomy" are:
  • cerebral ganglia (senses)
  • buccal ganglia (mouthparts)
  • pedal ganglia (foot)
  • pleural ganglia (mantle)
  • intestinal ganglia (organs)
  • visceral ganglia
See http://animals.about.com/od/mollusks/ig/World-of-Snails/Anatomy.htm.
Note that if you consider the ganglia that handle external senses and decides what action to take as the brain, then snails have one brain though in a snail that's stretching things somewhat. Note also that if you consider any significantly large interconnected cluster of nerves as a brain, then humans have four brains:
  • the large cluster in your head that you think with
  • the CNS comprising the spinal cord (which can easily be shown to have significant reflexes and some decision making capability - for instance the muscular cycle of walking arises in the spinal cord, not in your head) and part of what's at the top of it
  • the ganglia located in and controlling the heart
  • the coeliac ganglia controlling digestion. It is not commonly realised among people who are not medical specialists that the number of nerve cells in the human digestion ganglia is very large, and it can be shown by experiment to have considerable learning capability. For instance, if you are in the habit of eating 3 small/moderate meals 6 days a week, and a very large meal Sunday lunch time, and no evening meal for instance, the coeliac ganglia will set you up to optimally handle that. Try eating that Sunday roast for Wednesday breakfast and see how you get on. But if that's what you usually do, you'll be fine.
Wickwack 121.221.88.3 (talk) 14:39, 5 April 2013 (UTC)[reply]
Right. Nearly every animal has a nerve cord running the length of its body, with regularly spaced cell clusters called ganglia. The ganglion at the front is called the brain -- it is usually larger than the others and has special properties. Some types of worm also have an enlarged ganglion at the rear that is sometimes called a "tail brain". But that's about it, except when people speak euphemistically. Incidentally, our brain article covers all of this. Looie496 (talk) 14:50, 5 April 2013 (UTC)[reply]
If you are not satisfied with StuRat's description, see http://www.holysmoke.org/wb/wb0195.htm. Wickwack 120.145.202.43 (talk) 01:43, 6 April 2013 (UTC) [reply]

Chemistry

The oxidation state of chlorine in potassium chlorate is? — Preceding unsigned comment added by 41.220.68.33 (talk) 13:04, 5 April 2013 (UTC)[reply]

...a good question I ask my students for homework. Our oxidation state article might be a good place to learn how to figure it out.. DMacks (talk) 13:11, 5 April 2013 (UTC)[reply]

Universal life cycles

Is there scientific evidence that there are sun and moon-derived cycles that directly affect human behavior, yearnings and aspirations? If so, is this (in part) a result of our bodies sharing atomic structures common within the universe? — Preceding unsigned comment added by 98.248.153.47 (talk) 14:10, 5 April 2013 (UTC)[reply]

There is plenty of evidence of sun cycles affecting human behavior, but it has nothing to do with having "atomic structures in common within the universe," it has to do with the fact that pretty much all aspects of life on this planet are in some was affected by the amount of radiant energy that is coming from the sun (e.g. the weather). The moon is a convenient time-keeper and its movement affects the tides. But I don't think either of these are the kind of New Age sorts of explanations you are looking for. Just having atoms in common with something doesn't mean that you can mutually affect it. --Mr.98 (talk) 14:14, 5 April 2013 (UTC)[reply]
Lunar effect should cover it. Anecdotally, being related to nurses, it is a "Well Known Fact" that emergency admissions in the UK peak during the full moon. I'll try and find a news story related to it. --TammyMoet (talk) 17:51, 5 April 2013 (UTC) I knew I'd seen something about it relatively recently: this article summarises some research. --TammyMoet (talk) 17:59, 5 April 2013 (UTC)[reply]
You don't need to go for anything as unusual as the lunar effect. Of course the Moon has an effect on human behavior. For example, people think a full Moon is romantic and go out on dates around that time. If the Moon didn't exist, that wouldn't be the case. Also, people trip, fall, and hurt themselves during New Moon, but not as often during Full Moon. --140.180.248.141 (talk) 18:45, 5 April 2013 (UTC)[reply]
See also http://skepdic.com/fullmoon.html for a comprehensive summary. Also, the myth about a correlation between birth rate and moon cycle and/or tide has been clearly and repeatedly disproved. Cacycle (talk) 09:35, 6 April 2013 (UTC)[reply]
As a clerk in an ED, I can confirm nurses often blame busy shifts on "the full moon." Even when it's not a full moon. They've also decided that ordering Chinese food guarantees a bad day, and never say the word "code" or "trauma" because then we'll get one.
Now, most of the nurses aren't serious about it. It's just a simple way to vent stress at something that can't really be hurt. — The Hand That Feeds You:Bite 20:21, 7 April 2013 (UTC)[reply]
People also fish according to the lunar cycles, and some outdoor activites become possible at night only when that little bit of lunar illumination is availible, so that might affect the rates of emergency room visits too. 202.155.85.18 (talk) 03:55, 8 April 2013 (UTC)[reply]

Elephants and sign language

Our Article says elephants are really smart, like dolphins or smarter. Is it possible to teach an elephant to speak Sign language? (some modified version of it - using trunk). Like Koko the Gorilla or better.

Has someone tried to do it? --Zhitelew (talk) 14:27, 5 April 2013 (UTC)[reply]

I'm not aware of any definitive reference, but it would be very surprising if it were not possible to to teach signed symbol recognition, but the construction of a signed grammar is not likely possible. If you consult the reference cited in the Wikipedia article (Ref 5), it says: For 27 years she has lived among savanna elephants in Kenya's Amboseli National Park studying their behavior and methods of communication. Poole has found that the elephants use more than 70 kinds of vocal sounds and 160 different visual and tactile signals, expressions, and gestures in their day-to-day interactions, which is comparable to dogs. Being a dog owner myself, I can say from experience that it is easier to teach dogs (the large breeds at least) a comprehensive signed vocabulary for human-to-dog communication than it is to teach them spoken commands. And they certainly use a comprehensive set of expressions and body language to communicate back, should you care to pay attention (for example, ears down and walking to the side of you means "greetings, favorite/family person".
What primates can be taught, dogs cannot, and elephants probably cannot, is a grammar - i.e., "Give Koko fruit", or "No want fruit" as distinct from just "fruit" or "no".
When trying to evaluate the intelligence and communication abaility of animals, you need to remember two crucial things: a) emotions like anger, happiness, etc are universal to the animal kingdom, and are not unique to humans (contary to what some religious nuts would have you believe), and remarkably few neurons (like less than 10) are necessary to demonstrate a full range of emotions - as shown over 50 years ago by Gray Walter and other workers; b) there is something about the DNA of vertebrate animals that results in a brain volume or mass roughly in proportion to body surface area. A large animal needs are large brain to handle the stimuli and muscle control consequent to a large body. Humans are notable exceptions to this rule (known as the encephalisation quotient) - we have immense brains for our body surface area. Elephants most certainly do not. Not to be forgoten is that humans do much of their conceptual thinking very slowly - and that most likely makes our brain size go further. A large brain in a large animal such as a whale or an elephant does not necesarily mean high intelligence. The size is much dictated by the above rule, and much of it may be inefficiently utilised, or may be utilised for situational memory. Human brains need comparitively small volume for extensive lifetime memory because of our high conceptual powers. We don't remember the details of situations, we encode concepts and remember them.
Wickwack 121.221.88.3 (talk) 15:13, 5 April 2013 (UTC)[reply]
I am not sure why you attribute the denial of animal emotions to "religious nuts", when it is behaviouralists and doctors of an older generation who denied animals and newborns were conscious or experienced pain or actual emotions. This traditional denial by scienticians is all over the literature. μηδείς (talk) 17:36, 5 April 2013 (UTC)[reply]
Do you have a source? Also, who would you say is responsible for denying animal emotions today, as opposed to generations ago? --140.180.248.141 (talk) 19:38, 5 April 2013 (UTC)[reply]
Probably people who would feel guilty about what they do to animals, otherwise; like whalers, hunters, those who work in slaughterhouses or animal research labs, etc. StuRat (talk) 22:01, 5 April 2013 (UTC)[reply]
They tend not to be so articulate, passionate, and so noisy as those "deep south" religious types, though. Wickwack 120.145.202.43 (talk) 00:40, 6 April 2013 (UTC)[reply]
Any book you want to read on animal consciousness written in the last 20-30 years will do, like by Temple Grandin or Monty Roberts. Searle will give you good criticisms on behavioralism as applied to humans. Simple google searches on "do animal have emotions" and "do newborns feel pain" will get you everything you want. I don't understand the "who would you say is responsible" question. μηδείς (talk) 00:27, 6 April 2013 (UTC)[reply]

This is not a forum for posting a rant against behaviorist. Feel free to add a source, if you got any. OsmanRF34 (talk) 17:52, 5 April 2013 (UTC)[reply]

Emotion in animals has it all explained. Whether animals have emotions like happiness or not is far from uncontroversial. I suppose it's difficult to deny that mammals feel pain, however. OsmanRF34 (talk) 17:55, 5 April 2013 (UTC)[reply]

It would be hard to deny that a dog that is welcoming it's owner home after a week of absence with wild tail wagging, jumping, licking, etc isn't "happy". Sure it's possible that this is some bizarre behavioral thing that doesn't contain the underlying emotion - but how can you tell that your next door neighbor is "happy" and not just exhibiting some complex behavioral trick? SteveBaker (talk) 19:36, 5 April 2013 (UTC)[reply]
No, I cannot prove that my neighbor can feel something comparable to my feeling of happiness, and I don't deny that dogs have a whole plethora of feelings, and some of them could be called happiness. But how can I tell that a dog can have the same feeling of happiness which is not connected to any physical sensation, just caused to the realization that things are like you expect them to be? Some feeling in humans arise due what we know and to what information we get. Animals, contrary to my neighbor, process information in a quite different way, should I expect that they also feel some sort of existential anxiety like humans? OsmanRF34 (talk) 21:42, 5 April 2013 (UTC)[reply]
"Emotions" is kind of a stretch when applied to animals. As a pet owner, I'm more inclined to use the term "moods"; that is, is the animal feeling good, or not so good. ←Baseball Bugs What's up, Doc? carrots→ 22:24, 5 April 2013 (UTC)[reply]
Likewise, many animals also undeniably form interpersonal and social bonds, regardless of whether you interpret it from a strictly behaviourist perspective or consider it something akin to the attachment and affection we feel. Well, presumably we, since the existential question has been raised, but I for one accept it for a given...because, you know, I'm not a community college philosophy professor to dwell on the question of whether it might be otherwise - I've decided to get on with my life under the assumption that other people, and with some variation other creatures, have "genuine" emotions whose definitiveness is only limited by the same constraints I'd put on my own. But then, coming from a more contemporary (and biologically informed) perspective in the cognitive sciences, I tend to view the more absolutist approaches to behaviourism as more than a little flawed and antiquated. But we've surely strayed from the OP's point (even if we are barely still in the same nieghbourhood). Snow (talk) 08:32, 6 April 2013 (UTC)[reply]
Elephants already speak in sign language with their trunks and body gestures. They also vocalize, which anyone visiting a zoo at feeding time can see. The question is really "can elephants be taught to communicate with humans, in a human defined method of language?" Sure they can. Most animals can be trained to do many things. The thing they can't do, however, is to use those communication tools to express ideas, or to conceptualize, beyond the things that they were taught. Koko was so amazing because of the fact that she started to show sings of furthering her abilities on her own, but even that is still debated. As far as animals and emotions go, well, we can barely define the term happiness or sadness between two humans, much less different species. In my observations, most animals are either distressed, excited, or somewhere in the middle. I can assure you my dogs are "happy". Whether that happiness occasionally brings them a feeling of "joy" or not, I will never know, and I don't think anyone ever will. Ditch

biology

after fertilisation if uterus is removed from the body of female,what will happen? — Preceding unsigned comment added by Titunsam (talkcontribs) 17:19, 5 April 2013 (UTC)[reply]

Does the uterus still contain the fetus? If so, then the fetus will die. If the fertilized egg is, like, 17 years old, it's probably fine, though... --Jayron32 17:22, 5 April 2013 (UTC)[reply]
What will happen to the female? Or to the embryo/fetus/baby? OsmanRF34 (talk) 17:41, 5 April 2013 (UTC)[reply]
Also, for the sake of completeness, the removal of a uterus is called a hysterectomy. --Jayron32 20:39, 5 April 2013 (UTC)[reply]
I wonder, after re-reading the question, whether the OP meant: what happens after fertilization, when the uterus was removed? At first I thought that the ovaries are also removed during a hysterectomy (i.e. an oophorectomy was performed during a hysterectomy) , but that's not always the case. So, after removing the uterus, fertilization can result in rare occasions in an ectopic pregnancy.OsmanRF34 (talk) 21:57, 5 April 2013 (UTC)[reply]
I find it very hard to see how sperm can make the jump from the vagina to the Fallopian tubes, when all the ends are sewn up separately. They don't connect the vagina to the Fallopian tubes when the uterus is removed, assuming they leave them in place. The ovaries are not physically connected. --TammyMoet (talk) 10:46, 6 April 2013 (UTC)[reply]
It's very rare, but possible: WK has the explanation: "Removal of the uterus without removing the ovaries can produce a situation that on rare occasions can result in ectopic pregnancy due to an undetected fertilization that had yet to descend into the uterus before surgery. Two cases have been identified and profiled in an issue of the Blackwell Journal of Obstetrics and Gynecology; over 20 other cases have been discussed in additional medical literature." OsmanRF34 (talk) 13:22, 6 April 2013 (UTC)[reply]
Thanks for that - but just to make clear, except in situations where an undetected fertilisation was still in the Fallopian tube, pregnancy from unprotected sex cannot happen after hysterectomy. --TammyMoet (talk) 13:59, 6 April 2013 (UTC)[reply]
That makes a lot more sense, but is it possible that they could botch the operation and somehow a sperm could find its way to the right place? ←Baseball Bugs What's up, Doc? carrots→ 15:24, 6 April 2013 (UTC)[reply]
I don't see how. Even if somehow they failed to seal both the vagina and an ovary, there's no "path" for the sperm to follow. Further, sperm has a very low lifespan after it exits the male body. Chances of one being able to slog along other tissue to reach the exposed ovary are virtually zero.
Also, the woman would be experiencing some pretty nasty bleeding if they failed to seal the vagina. — The Hand That Feeds You:Bite 20:40, 7 April 2013 (UTC)[reply]

The lifetime of an alpha particle compared with that of the proton

ive heard the [free or otherwise] proton may decay, with a very long halflife like 10^40seconds. Now the free neutron lasts justs a few seconds. Well, could alpha paricles be even more stable than protons, just as neutrons seem to last longer (or "pass the hot potato" to another nucleon when about to decay??) inside most nuclei? Thanks, Rich.199.33.32.40 (talk) 20:14, 5 April 2013 (UTC)[reply]

AFAIK, the alpha particle is essentially stable over any arbitrarily long time span. Helium-4 discusses the stability of the particle some, and the He-4 nuclide is noted on all of our various lists of nuclides (including List of nuclides and List of elements by stability of isotopes) as not having a known half-life, which would count as "about as stable as can be found". --Jayron32 20:30, 5 April 2013 (UTC)[reply]
Unlike the the Neutron which may decay more slowly when found within a nucleus, The proton's lifetime really isn't affect by it's environment (except in the case of a beta plus decay in which the proton decays into a neutron,m but that's a different matter entirely. Dauto (talk) 19:27, 6 April 2013 (UTC)[reply]
If the lifetime of a lone proton is finite, alpha particles will decay by the same process with a slightly shorter half life (since any one of the four nucleons can decay). Alpha particles should also be subject to sphaleron decay, unlike lone protons, but the half life for that is ridiculously long. In a recent thread Count Iblis said the half life for sphaleron decay of a deuteron is 10218 years, but I don't know where he got that figure. Either way, alpha particles definitely have a shorter lifetime than protons. -- BenRG 05:53, 7 April 2013 (UTC)
So is there no possibility that the halflife of the protons are lengthend by their inclusion in the alpha particle, by analogy to neutron's halflife being lengthened by being in a nucleus? 202.155.85.18 (talk) 03:50, 8 April 2013 (UTC)[reply]
Thanks to all.-Richard Peterson76.218.104.120 (talk) 03:27, 9 April 2013 (UTC)[reply]


April 6

Can someone, please, correct this messed up navbox?

There is an obvious error in navbox in article Stainless_steel headed: "Iron-carbon alloy phases", where it lists "Graphite (allotrope of carbon)", as a phase but that is erroneous:

That navbox element is incorrect because graphite is formula C, and is not a phase of steel alloy. In fact graphite is a form of pure carbon, and is of no more relevance to stainless steel phases than diamonds would be... Garshepp (talk) 04:04, 6 April 2013 (UTC) — Preceding unsigned comment added by Garshepp (talkcontribs) 04:01, 6 April 2013 (UTC) Garshepp (talk) 04:04, 6 April 2013 (UTC)[reply]

It IS a phase in pig iron cast iron, and the infobox is about ALL phases in ALL iron-carbon alloys, not just steel. I thought pig iron and cast iron were the same thing -- apparently not. Live and learn!24.23.196.85 (talk) 04:17, 6 April 2013 (UTC)[reply]

Tire pressure sensors

Last night after I put air in my tires because the light on my dashboard told me to, I started wondering how the car knew. I haven't found a clear explanation of the mechanics of it. So, could someone educate me? Where are the sensors and how do they test the pressure? Thanks, Dismas|(talk) 11:57, 6 April 2013 (UTC)[reply]

See tire-pressure monitoring system. The sensor is inside the tire, around the valve stem (there's an image here: www.exam iner.com/article/tire-pressure-monitoring-it-s-not-a-whole-bunch-of-hot-air-it-s-the-law spam filtered, so manually fix that link yourself). That monitors the pressure (the pressure differential across the valve) and communicates by radio back to a module inside the car. -- Finlay McWalterTalk 12:14, 6 April 2013 (UTC)[reply]
Actually, there are two common methods. One is the direct measurement technique that Finlay describes - but the older (and much cheaper) method is to have the computer count the number of times each wheel rotates and if (on average) one is rotating faster than the others, then its circumference must be a little less - which is most likely due to the pressure being lower. SteveBaker (talk) 19:39, 6 April 2013 (UTC)[reply]

Stars

If a red dwarf crosses paths with a white dwarf, which one is more likely to capture the other? 24.23.196.85 (talk) 19:40, 6 April 2013 (UTC)[reply]

A red dwarf usually has less mass, so it would be deflected more. However, they would each capture each other, meaning they would orbit a barycenter between the two, but closer to the white dwarf. Note, however, that there is some overlap of solar masses, ranging from 0.075 to 0.500 for a red dwarf and from 0.140 to 1.330 for a white dwarf, so, while I described the average pairing, you could have pairs of equal or even the reverse mass ratios. StuRat (talk) 19:47, 6 April 2013 (UTC)[reply]
Thanks! 24.23.196.85 (talk) 20:24, 6 April 2013 (UTC)[reply]
You're quite welcome. Can we mark this Q resolved ? Also, you might want to check out Pluto and Charon (moon), which have a similar mass ratio and thus orbit a common barycenter. StuRat (talk) 21:08, 6 April 2013 (UTC)[reply]

The following discussion is closed. Please do not modify it. Subsequent comments should be made on the appropriate discussion page. No further edits should be made to this discussion.


StuRat, can you please refrain from answering questions on subjects you know little about, which apparently includes fundamental physics? Your answer is almost completely wrong.
If a red dwarf and white dwarf cross paths, unless there's a third body, they will not capture each other. This is because the two bodies would have enough kinetic energy to escape each other's gravitational pull. In Newtonian gravity with 3 bodies, there are 2 situations that can happen:
1. The two bodies have too much energy. In this case, they don't orbit each other, but escape each other's gravity and keep on going. Their trajectories describe a hyperbola.
2. The two bodies have too little energy. In this case, the bodies orbit each other in an ellipse whose shape and size never changes.
Because energy is conserved, you can't go from 1 to 2 or vice versa unless some third body robs the system of energy. This is what happens during gravitational capture. However, the 3-body problem has no analytic solution and is quite hard to analyze, so there's no simple description of what happens during gravitational capture.
StuRat is correct, however, in implying that it's meaningless to say whether one body captures the other. The Moon doesn't orbit Earth; it orbits a common barycenter in between the Earth and Moon, but much closer to Earth. --140.180.248.141 (talk) 23:02, 6 April 2013 (UTC)[reply]
With stars, you have more complex motions than just 2 or 3 bodies. They are also orbiting the galactic center, and there are gravitational waves affecting their motion within the arms. And, there also appears to be a dark matter effect on them, as their motion can't completely be explained without it. So, the actual capture process would be quite complex. I chose not to describe possible capture processes, since the question didn't seem to be about that. As I read it, they were only asking what the eventual orbits would be, IF a capture did occur. Now, please desist with the insults. They do not belong on this page (or anywhere, really). StuRat (talk) 23:15, 6 April 2013 (UTC)[reply]
So, if a red dwarf were to collide with a white dwarf, would the result be a pink dwarf? ←Baseball Bugs What's up, Doc? carrots→ 23:36, 6 April 2013 (UTC)[reply]
No, the result would be a larger red dwarf. Whoop whoop pull up Bitching Betty | Averted crashes 00:07, 7 April 2013 (UTC)[reply]
The effect of the galactic center, gravitational waves, and dark matter are utterly negligible when considering an encounter between two stars. LIGO failed to detect any gravitational waves, and it can measure a 4 km length to within 10^-18 m (about 1000 times smaller than a proton). Dark matter and the galaxy accelerate both bodies by almost exactly the same amount. Only the tidal force can have any effect, and it's negligible for galactic distances.
There are other effects that influence gravitational capture. For example, tidal friction induced by one body on the other can make the system lose energy. The red dwarf could lose material to the white dwarf; this process is called accretion. If enough material is accreted, the white dwarf could explode in a Type Ia supernova.
I don't think I insulted you. I said you don't seem to know fundamental physics. I stand by that judgment, and would add that you don't seem to know much about astrophysics either. --140.180.248.141 (talk) 00:01, 7 April 2013 (UTC)[reply]
And do you believe such statements belong here ? Shall I start insulting you ? You've just listed a couple scenarios where a capture could take pace, but, as I said previously, I don't see this Q as being about the capture processes, but only the eventual result of the capture. Where do you see anything in the Q requiring us to describe all possible capture processes ? And, even if it had asked for that, then my answer would be a partial one, not an incorrect one. I just don't see what on Earth you're bitching about. A more constructive way for you to respond would have been: "While StuRat described the eventual result of such a capture, I'd also like to discuss possible processes whereby such a capture might occur...". StuRat (talk) 00:48, 7 April 2013 (UTC)[reply]
You said "however, they would each capture each other". In actuality, they would pass each other without incident unless a third body was involved, because gravitational capture is impossible with only 2 bodies. You said that "the actual capture process would be quite complex" and implied that the galactic center, gravitational waves, and dark matter make it complex. In actuality, all three have an utterly negligible effect. --140.180.248.141 (talk) 01:44, 7 April 2013 (UTC)[reply]
The OP did not specify a universe containing only the two stars. That's your assumption. So, with other stars in the neighborhood, they may have a significant gravitational effect, especially on the scale of billions of years, with orbital distances of multiple light years. (The time frame and distances for the capture was also not specified.) And, over those time scales, the other factors I mentioned, like dark matter and gravitationaly waves, may also have a significant effect on the motion of the stars. StuRat (talk) 02:22, 7 April 2013 (UTC)[reply]
However accurate or otherwise another editor's comments might be, it is not appropriate to comment on or express a sentiment on their level of knowledge, nor on the advisability of their contributing, when such contributions are in good faith, as StuRat's clearly were. Please confine comments to the subject matter; corrections to another editor's contributions are quite sufficient in the context. — Quondum 02:08, 7 April 2013 (UTC)[reply]
Agreed. StuRat (talk) 02:22, 7 April 2013 (UTC)[reply]
Disagreed 100%. In the context of trying to communicate in a forum such a this, pointing out when another editor is beyond their ken is entirely germane, if done in a simply factual matter. I don't see any insult here under any definition. --Mr.98 (talk) 03:02, 7 April 2013 (UTC)[reply]
"Can you please refrain from answering questions on subjects you know little about, which apparently includes fundamental physics?" is not a "factual" statement, it's a gratuitous insult that serves no purpose other than to inflame. If the snippy IP had simply said, "That's not correct", and then explained why, it could have been a different story. ←Baseball Bugs What's up, Doc? carrots→ 06:38, 7 April 2013 (UTC)[reply]
Sturat's first answer was correct, on the obvious assumption the OP wanted cases where the two stars were moving slowly enough to become gravitationally entangled. IP 140 is simply wrong that the barycenter of the earth-moon system lies between the two bodies--it lies ~1000 miles below the earth's surface. μηδείς (talk) 18:39, 7 April 2013 (UTC)[reply]
Barycentric coordinates (astronomy) has the confirmation to Medeis. I wonder where 140.180 studied physics, if at all. His IP geolocated to Princeton, but I hope they don't have such a low level. Not knowing this is not scary if you are not into natural sciences. However, accusing StuRat of knowing little fundamental physics and then making such a mistake is laughable. OsmanRF34 (talk) 20:03, 7 April 2013 (UTC)[reply]
The barycenter is between the centers of the two bodies. The center of a spherical body is usually used for gravitational calculations, and never did I imply that the barycenter is outside Earth's surface. Contrary to what Medeis seems to imply, it is not possible for two bodies to become "entangled" (meaning orbiting a barycenter) in the absence of a third body, no matter how slowly the stars are moving. This is a common misconception about gravitational capture, and since the OP seemed to hold it, people answering this question are responsible for correcting it. Failing to correct it is similar to answering "how many tons of green cheese is the Moon made of?" with the mass of the Moon, without also saying that the Moon is not made out of green cheese. (Formerly 140.180.248.141) --128.112.25.104 (talk) 04:16, 8 April 2013 (UTC)[reply]
The discussion above is closed. Please do not modify it. Subsequent comments should be made on the appropriate discussion page. No further edits should be made to this discussion.
Is it possible to capture by aerobraking? With low enough masses, they don't have to cause a type Ia supernova. Well, to tell the truth, in a flat, non-expanding classical Newtonian 2-body universe, even if the stars were atmosphereless spheres that won't touch or change (i.e. subliming into the vacuum), passing does not guarantee that they won't pass again. They can't be infinite distance now can they, there's always a higher real number. No matter the distance a closed orbit is simply a matter of low enough speed for the distance, but not so low that they collide). Sagittarian Milky Way (talk) 18:27, 8 April 2013 (UTC)[reply]

April 7

Inflammatory Diseases Caused by Defect in the TLR Pathway

Hello. Would someone please list an inflammatory disease caused by a defect in the Toll-like receptor pathway? Thanks so much in advance. --Mayfare (talk) 01:59, 7 April 2013 (UTC)[reply]

Please do your own homework.
Welcome to the Wikipedia Reference Desk. Your question appears to be a homework question. I apologize if this is a misinterpretation, but it is our aim here not to do people's homework for them, but to merely aid them in doing it themselves. Letting someone else do your homework does not help you learn nearly as much as doing it yourself. Please attempt to solve the problem or answer the question yourself first. If you need help with a specific part of your homework, feel free to tell us where you are stuck and ask for help. If you need help grasping the concept of a problem, by all means let us know. --Jayron32 02:48, 7 April 2013 (UTC)[reply]

Hello Jayron. I have done some research. I could not find what I was looking for. (Instead, I found sources about beneficial mutations.) I posted half of my question so I can have better search terms. After then, I will write my essay on my own. I graciously ask for your understanding. --Mayfare (talk) 04:23, 7 April 2013 (UTC)[reply]

It sounds as though research is an essential part of the assignment. Searching Pubmed for "tlr AND autoimmune" produces many interesting results - the top 5 of which include direct answers to your question. -- Scray (talk) 13:46, 7 April 2013 (UTC)[reply]

viroids

   what are viroids?
   a)infectious nucleoprotiens
   b)infectious nucleic acid
   c)infectious proteins
   d)infectious lipoproteins  — Preceding unsigned comment added by 117.227.142.217 (talk) 02:50, 7 April 2013 (UTC)[reply] 
We won't answer homework problems, but you should know that the answer to your question is in our articles:
what are viroids?
   a)infectious nucleoprotiens
   b)infectious nucleic acid
   c)infectious proteins
   d)infectious lipoproteins
it's often a good start just to type the words in the question into the search box. SemanticMantis (talk) 03:05, 7 April 2013 (UTC)[reply]

Are there any planetariums where the dome screens touch the floor?

Because some planetariums' dome screens don't reach the floor. Having a dome screen that touches the floor is better because it's more realistic that way. Because if you're outside, it looks like the night sky touches the ground, and that's the kind of experience I'm looking for in a planetarium. Therefore, the planetarium needs to have a dome screen that touches the floor. Are there any planetariums like this that exist? Mattdillon87 (talk) 07:12, 7 April 2013 (UTC)[reply]

Wouldn't the people in the audience get in the way of the projected images? ←Baseball Bugs What's up, Doc? carrots→ 07:19, 7 April 2013 (UTC)[reply]
Yes there is. And it's portable too! --TammyMoet (talk) 10:24, 7 April 2013 (UTC)[reply]

Source

I need a source for a statement at the behest of another. The statement is: "There are no commercial sources for azanes." This intends to say that there are no naturally occuring reserves of azanes, which are commercially exploited. Azanes include ammonia, Hydrazine, etc. Plasmic Physics (talk) 09:42, 7 April 2013 (UTC)[reply]

That's a very unusual meaning of "commercial sources". By that definition, there are no commercial sources of aluminium or sulphuric acid. I don't think the statement can really be substantiated. Tevildo (talk) 12:00, 7 April 2013 (UTC)[reply]
Bauxite doesn't contain 'aluminium' per se, but it does contain various aluminium oxides and hydroxides, which must be processed to obtain aluminium actual. Plasmic Physics (talk) 12:30, 7 April 2013 (UTC)[reply]

Addendum: To me it seems an obvious statement in need of no citation according to WP:BLUE, based on my inability to find sources supporting a contrary position. Plasmic Physics (talk) 10:02, 7 April 2013 (UTC)[reply]

Ammonia can be made from urine, a lot more easily than aluminum can be made from bauxite. It's unclear to me whether that would count as a "commercial source" in this context. Looie496 (talk) 16:01, 7 April 2013 (UTC)[reply]
Linking to the article: azane. Said statement is at the first paragraph, last sentence. OsmanRF34 (talk) 16:17, 7 April 2013 (UTC)[reply]
First off, the statement "There are no commercial sources for azanes" is just simply bad, even if it's referenced and true under whatever particular definition of "commercial sources" you're using. It isn't clear just from that statement what that definition you're using here is, so someone else reading it will not understand your meaning. Rephrase to be more precise and clear. (Finding references might depend on what that rephrasing is. I'm still not sure what the meaning is intended to be.) - I also think WP:BLUE doesn't quite apply here. It's applies to things which are common knowledge. The "commercial sources" of azanes is far from common knowledge. If the statement regarding it is noteworthy enough to be stated as it is now, it probably should be substantiated. (Especially since the statement is highly dubious. (What do you mean by "no commercial sources"?) "Inability to find sources on the contrary position" doesn't cut it as justification, especially when no sources can be found for the collaborating position. There's a whole host of statements one could make that have no sources for or against. Only a small subset would actually be true on closer examination. -- 71.35.122.64 (talk) 20:51, 7 April 2013 (UTC)[reply]
Before we go further, what is the interpretation of "commercial sources" in the statement featuring the term, in the introductory paragraph of alkane. My intention is to use the same definition, but apply it to Azane. Plasmic Physics (talk) 22:12, 7 April 2013 (UTC)[reply]
The alkane statement, for reference, is "There are two main commercial sources: crude oil and natural gas.". So, the main commercial sources for ammonia are air and methane (see Haber-Bosch process), and the main commercial sources for hydrazine are air, methane and brine (see Olin Raschig process and sodium hypochlorite). Tevildo (talk) 22:22, 7 April 2013 (UTC)[reply]
How about "All commercially available azanes are synthetically derived"? 202.155.85.18 (talk) 03:23, 8 April 2013 (UTC)[reply]
That could work. Plasmic Physics (talk) 07:31, 8 April 2013 (UTC)[reply]

Check these facts

I have found some unbelievable facts. Four of them are as following -
Please check whether the following facts are right or wrong
1. Some species of earthworm can have as many as 10 hearts.
2. Space expand faster than the speed of light.
3. A bird’s heart beats 400 times per minute while they are at rest. When they are flying however, their heart could beat up to 1000 beats per minute.
4. Electricity travels at the speed of light.Scientist456 (talk) 12:48, 7 April 2013 (UTC)[reply]

1. Earthworm#Circulatory_system: "Segments in the esophageal region contain muscular commissural vessels connecting the top and bottom vessels that function like hearts to pump the blood (these are called aortic arches). There are five pairs of hearts, more or less."
2. Metric expansion of space: "While special relativity constrains objects in the universe from moving faster than the speed of light with respect to each other, it places no theoretical constraint on changes to the scale of space itself. It is thus possible for two objects to be stationary or moving at speeds below that of light, and yet to become separated in space by more than the distance light could have travelled, which can suggest the objects travelled faster than light."
3. We don't have an article on that, but small hearts beat very fast, it's true. We do have a piece on Hummingbird#Metabolism: "With the exception of insects, hummingbirds while in flight have the highest metabolism of all animals, a necessity in order to support the rapid beating of their wings. Their heart rate can reach as high as 1,260 beats per minute, a rate once measured in a Blue-throated Hummingbird." Perhaps as amazing, at night they can slow their heart rate to 50-180 beats per minute as part of a hibernation-like state.)
4. Speed of electricity: "The word "electricity" refers generally to the movement of electrons (or other charge carriers) through a conductor in the presence of potential and an electric field. The "speed" of this flow has multiple meanings. In everyday electronics, the signals or energy travel quickly, as electromagnetic waves, while the electrons themselves move slowly." — AKA it's complicated. --Mr.98 (talk) 13:02, 7 April 2013 (UTC)[reply]
It is not our purpose here to enjoy a miscelany of odd facts. Our purpose here is to answer questions about things which you do not know or understand, but would like to. We ask that you do your own homewark first, then post a question if & when yiou get stuck. However, taking your numbered items in turn:-
1. Googling "earthworm anatomy" reveals a host of websites that show that earthworms don't actually have any true hearts. The dorsal blood vessel contracts to push blood about. If, by "heart", you mean the aortic arches, which assist in moving blood to the capillaries, then yes, there are typically 5 pairs of arches.
2. I'll leave that to those knowlegeable in the subject for a definitive answer, but suspect that this too is a misunderstanding of the facts.
I'm no expert, but from what I understand, #2 is a key aspect of the theory of inflation (cosmology) used to explain away the horizon problem. Clarityfiend (talk) 03:39, 8 April 2013 (UTC)[reply]
3. Large birds such as pelicans and albatrosses have pulse rates much the same as humans. I've held chickens in my hands and felt their heart beating through their chest. They run a little fast but nothing like 400 bpm. Small birds such as pidgeons and parrots run about 160 (rest) to 500 (maxium effort climbing flight) or so. Birds meet their high oxygen requirments for flight by having comparitively larger hearts and higher blood pressure for their body mass, compared to mammals.
4. Electricity certainly does not travel at the speed of light. How fast electrical energy propagates down a wire or cable depends on the distributed inductance and capacitance, and for typical cables is around 0.5 to 0.7 x the speed of light. Note that the speed that electrical energy travells IS NOT the speed at which electrons move.
Wickwack 58.169.234.119 (talk) 13:20, 7 April 2013 (UTC)[reply]
Off-topic metadiscussion
The following discussion has been closed. Please do not modify it.
DFT, FFS. And please don't tell me we should treat questions in good faith, That is how the Nigerian scam worked. This question(s) are clearly bait. Caesar's Daddy (talk) 14:40, 7 April 2013 (UTC)[reply]
I have no idea what your contribution is all about. OsmanRF34 (talk) 15:51, 7 April 2013 (UTC)[reply]
Well, Gaius might be converting a finite list of equally-spaced samples of a function into a list of coefficients of a finite combination of complex sinusoids, ordered by their frequencies, or might be considering a dynamic - cognitive approach to human decision making, or might be referring to a quantum mechanical modelling method used in physics and chemistry to investigate the electronic structure (principally the ground state) of many-body systems, in particular atoms, molecules, and the condensed phases, or any one of a choice of interpretations of his first TLA, but I don't think the second one refers to reconstructive surgical procedures that alter typically male facial features to bring them closer in shape and size to typical female facial features. Does this clarify things? I don't understand, either, and I wish people wouldn't assume that everyone else uses the same abbreviations. Dbfirs 16:28, 7 April 2013 (UTC)[reply]
WP:DFT was intended, unjustifiably IMO. Tevildo (talk) 16:41, 7 April 2013 (UTC)[reply]
Ok, don't feed the trolls, for fuck's sake. Now I get the message, but still doesn't know what's wrong with the series of questions above. They seem to be general curiosity. OsmanRF34 (talk) 16:54, 7 April 2013 (UTC)[reply]
(edit conflict)Thanks, that solves one mystery. Quote from that redirected link: "If a user seems to be asking stupid questions, try to give them the resources to help themselves." I think the replies to the seven questions have been directed towards that aim, and I found the questions basic, but more interesting than disruptive. Dbfirs 17:03, 7 April 2013 (UTC)[reply]

24k Silver

If gold and silver are about the same hardness, why doesn't 24k silver exist for jewellery?Curb Chain (talk) 15:37, 7 April 2013 (UTC)[reply]

The carat only applies to gold - for silver (and precious metals in general), the measure used is Millesimal fineness. According to that article, the finest silver commonly used for jewellery is Britannia silver at 958. Tevildo (talk) 15:56, 7 April 2013 (UTC)[reply]
That's the same purity as 23karat. CS Miller (talk) 16:08, 7 April 2013 (UTC)[reply]
I think there is some confusion to my question: What I attempted to ask is "Gold jewellery exists in 24k. Why doesn't silver jewellery exist as widely at this purity?".Curb Chain (talk) 02:11, 8 April 2013 (UTC)[reply]
Because it's more reactive, and harder to purify (also less valuable, which means there's not as much need for ultra-high purity). 24.23.196.85 (talk) 02:19, 8 April 2013 (UTC)[reply]
Is silver that much more harder to purify that there is no demand for pure silver jewellery?Curb Chain (talk) 10:45, 8 April 2013 (UTC)[reply]
"Pure" silver is less suited to many applications because it is softer and has a higher melting point than sterling silver and other alloys. DMacks (talk) 02:23, 8 April 2013 (UTC)[reply]

East

East Africans are better at long-distance running while west-Africans are better at short-distance sprinting. Can this be attributed to evolution? Pass a Method talk 19:03, 7 April 2013 (UTC)[reply]

note The winners of last years Olympic long distance events (800m, 500m 10,000) were of East African descent. The winners of short-distance events (100, 200m, 400m) were all of West-African descent. Pass a Method talk 19:14, 7 April 2013 (UTC)[reply]
According to [[1]], Kenyans at least are not better long-distance runners due to their genes. OsmanRF34 (talk) 19:55, 7 April 2013 (UTC)[reply]
(ec) My gut feeling is that this has to do with fast-twitch and slow-twitch muscle fibers. A quick Google search confirms [2] Poking NCBI the same way I get PMID 17087108 and PMID 15286361 .. apparently there can also be a downside, in obesity stats [3]. I didn't generally sort out the details of which populations and genes are involved (a large task) but it looks like alpha-actinin 3 has a special role, being completely absent from a billion individuals but being absent in only 6% of sprinters vs. 18% of the general population. [4]. However, a study of it [5] didn't find that it had an effect on the number of sprinters, because it is present at only 1-11% in the African populations they looked at. I expect you'd have to tot up many such genes and studies to get to the truth... perhaps there's a review I missed... Wnt (talk) 20:03, 7 April 2013 (UTC)[reply]
There's enough genetic diversity within African, American, Asian or European populations to find enough individuals of any genetic markup, equivalent to what you expect to find in an Olympic runner. The key difference is how many individuals would ever try to become good at running in a specific geographical area. Long-distance running is very popular in Kenya. And it is by no means a fact of life that Eastern African are the best long-distance runner. If my memory doesn't fail me, in the past Finland and Hungry were overrepresented. In the same way that Norwegians are overrepresented at biathlon or Romanian at weight-lifting. OsmanRF34 (talk) 20:08, 7 April 2013 (UTC)[reply]
(ec) The flip answer is "I think you just did."
The slightly longer answer is "Yes, but that's probably not the question you meant to ask." That is, individual sources which have attributed the difference to evolutionary pressures are readily discoverable through a cursory Google search. What I'm guessing you're really interested in is whether or not those conclusions are true. There exists lots of evidence supporting a genetic basis for the dominating superiority of East Africans (at least, individuals of East African genetic heritage) in long-distance running, and West African supremacy in sprints. Probably the best-known popular treatment of this topic is Jon Entine's Taboo: Why Black Athletes Dominate Sports and Why We’re Afraid to Talk About It, though it may be a bit dated since it was published in 2000.
That gets us to the meatiest but probably least satisfying answer. While it's tempting to say (post hoc ergo propter hoc) that every genetic result in biology is driven by evolutionary (selection) pressure, you can also get some weird coincidences and counterintuitive outliers. Suppose there was an isolated East African population that coincidentally carried genes for distance running, which have been passed down through some sort of founder effect? Some have suggested that this is indeed a big part of East African distance running success—many marathoners are drawn from a a single tribe, the Kalenjin people of the Rift Valley Province of Kenya, [6]. Alternatively or relatedly, as the likely origin of the human species and home to the oldest human population, Africa is also home to the greatest human genetic variation—more genetic mixing and matching and a more 'complete' set of founders means more opportunity to coincidentally come up with the most effective 'sprinter' and 'runner' phenotypes.
Is there something intrinsic to the environment of the Rift Valley that would apply a particular selection pressure in favor of distance running? Or in favor of sprinters in West Africa?> The jury's still out, as far as I can tell, but there doesn't seem to be any compelling argument or evidence for it so far: [7].
Study is further complicated or confused by an overwrought (and sometimes reflexive) accusation that looking for genetic factors contributing to these atheletes' abilities is inherently racist, and that scientists who examine these questions are somehow attempting to dismiss the hard work and training of African athletes as nothing more than a well-he-was-just-born-that-way genetic lottery ticket [8]. TenOfAllTrades(talk) 20:14, 7 April 2013 (UTC)[reply]
I'd say theres definitely a genetic element to this, because you see the same pattern in immigrant communities as in native communities. In other words, both East African immigrants and East Africans themselves are great long-distance runners. Same pattern with West Africans at 100m. Pass a Method talk 20:35, 7 April 2013 (UTC)[reply]
Sure; I'm sorry if I didn't make that clear. I'd say that the general consensus is that there is a (strong) genetic factor to the success of (East|West) Africans in (long|short)-distance running. The open question is whether or not this is a fluke or if it is due to an identifiable selection pressure. TenOfAllTrades(talk) 22:35, 7 April 2013 (UTC)[reply]
http://journals.cambridge.org/action/displayAbstract;jsessionid=73C786F70EF0911ED6A9BD665E87A4E7.journals?fromPage=online&aid=793784
http://news.bbc.co.uk/sport2/hi/olympic_games/world_olympic_dreams/8886705.stm
--Guy Macon (talk)
As said in the article linked by Guy Macon: "however, any genetic component to their success is unlikely to be limited to East Africans and is more likely to be found in other populations. At present it is unjustified to implicate a role for genetics in the success of East African runners when no genes have been identified as being important to their performance." No matter what genes are good for it, if any, you'll find them elsewhere. Americans are dominant in baseball. Spaniards are good in bull-fighting. Swedish and Canadians in curling. Japanese in Sumo. Is that all genetically motivated? Note: TenOfAllTrades' linking to Forbes and Guardian as sources and claiming that there is broad consensus is kind of funny. OsmanRF34 (talk) 01:02, 8 April 2013 (UTC)[reply]
Ten gave a good answer. I should explain that the reason why looking for a genetic edge, even a racial genetic edge, is more plausible in these competitions than in normal life is that we are measuring extremely small differences and doing so in a very small subset of the population. If two races have a hundredth of a point difference in their IQ scores, it will not be detected nor is it grounds for discrimination, but if they have a hundredth of a second difference in running time of their very fastest sprinters, it could really matter. These tiny differences also matter only under the probably reasonable assumption that all the best athletes are putting everything they possibly can into their training and competitions. Even so - nobody is going to be able with current technology to point at an athlete and say for sure that his genes won the race; it is at best a statistical hypothesis, and rarely developed far enough for that. Wnt (talk) 04:23, 8 April 2013 (UTC)[reply]
The assumption "that all the best athletes are putting everything they possibly can" is not necessarily true. Americans and Europeans who are into sports have plenty of options, and therefore many who would be good runners end in different sports, playing at a high level. The question I asked above stays in the room, for all that believe that the gens of a population made them be better at something: Are Swedish and Canadians better in curling due to their genetic predisposition? Even if you found that the runners have different genes (which is pretty probable, they must at least be thin and not too tall, in the case of marathon runners), that wouldn't mean that the East African population can produce more runners than say the Czech population. OsmanRF34 (talk) 16:42, 8 April 2013 (UTC)[reply]


There was a BBC Horizon documentary about this a long time ago. They concluded that what makes East Africans better at long distance running is that certain leg bones are longer, making running more efficient for them. They tested this by monitoring the progress of British and Kenyan boys at running. Their fitness and running performance was monitored over time as they exercised. What became clear is that for the same value of indicators of fitness like VO2 max etc., the Kenyans would run faster. Count Iblis (talk) 17:42, 8 April 2013 (UTC)[reply]

I wonder what scientific studies are there, which are not in Forbes or BBC or Guardian, about the relationship between genetics and African runners. If you pick some fatty English boys for a comparison, then sure, the Kenyans will run faster. Can we say that this or that population is better at being the runner elite? No, even if we know that height and fat are partially genetic determined. OsmanRF34 (talk) 19:41, 8 April 2013 (UTC)[reply]
If the BBC documentary was right, then it is mainly an issue of biomechanics. It may be that there are many genes involved so you don't really have clean genetic signal from DNA analysis. However, if biomechanical experts link it to the typical East African bodybuild, then you don't need to study the DNA. I'm sure that there are peer reviewed scientific studies that make this point. In case of the 100 meters sprint, while the biology is less clear here, the data points to an even larger disadvantage for Europeans. There is only one white person who has ever run the 100 meters in less than ten seconds.
This same biomechanics that allows Kenyans to run more efficiently than Europenas has also been invoked to explain why Neaderthalers died out. Apparently the body build of Neaderthalers made them a lot less efficient to walk or run large distances than Homo Sapiens. They used to hunt animals by ambushing them and killing them from close range. Due to environmental changes the dense forests disappeared making hunting this way less efficient compared to trail seeking and stalking animals which involves covering large distances as our hunter gatherer ancestors used to do. Count Iblis (talk) 23:32, 8 April 2013 (UTC)[reply]
aside from the fast twitch slow twitch thing, there's differences in the mechanical design of a sprinter vs a marathoner, similar to the reasons why a top fuel dragster can get along without a radiator but a le mans car can't, for instance. http://naturallyengineeredcom-files.s3.amazonaws.com/blog/wp-content/uploads/2012/08/olympic-sprinters-body-infographic.jpg and http://naturallyengineeredcom-files.s3.amazonaws.com/blog/wp-content/uploads/2012/08/olympic-marathon-body-infographic.jpg. Gzuckier (talk) 06:10, 9 April 2013 (UTC)[reply]

Effect of galvanisation on steel used as anode in electrolytic rust removal

I would like to use electrolytic rust removal. Can galvanised steel be used for the anode? Would this have any effect on the process or finished piece? 89.241.233.102 (talk) 20:57, 7 April 2013 (UTC)[reply]

Specifically, I want to restore this vice. Would it matter if it got some zinc plating on it? Would it interfere with the rust removal? --89.241.233.102 (talk) 21:10, 7 April 2013 (UTC)[reply]
My suggestion is, first to strip it apart. Apply tape to the screw and nut -to protect it. Then de-rust parts in a weak 3 ½ % solution of hydrochloric acid (spirit of salts). Then dry and paint with a good quality enamel paint. Electrolytic Rust Removal is great for maybe archaeological exhibits but it only goes part way to restoring the original morphology but no way to restoring the granular structure of the steel itself. If you use electrolysis for this application you will just end up with crud in the nut and it will soon seize up. --Aspro (talk) 21:50, 7 April 2013 (UTC)[reply]
Thanks a lot for your advice! How and why do I need to apply tape? Why aren't archaeological exhibits de-rusted in hydrochloric acid? What do you think is the most economical form for me to buy hydrochloric acid in? I'm in the UK, if that makes a difference. I can get it on eBay but maybe there's somewhere local, if I know where to look. A hardware store? An arts store? — Preceding unsigned comment added by 89.241.233.102 (talk) 23:02, 7 April 2013 (UTC)[reply]
Archaeological finds are mostly iron oxides. If the conservationists removed all that crud with acid, there would be little left. Electrolytic Rust Removal reverses the process back to something resembling the original form. However, the object is still very delicate. So, say a Spanish cannon, after years on the sea-bed, was so thus restored -it would likely explode if it it was fired again. It may look good but the original crystal structure (which gave it its tensile strength) would not have reformed. The average guy can readily buy hydrochloric acid in the UK as 'spirit of salts' (sold as a drain cleaner) from DIY stores, high street Chemists, etc. (locally it only cost about £2 for 500ml). The bottle will state its strength. From this one can work out how much water you should add it to. Note: All ways pour acid into water.. not add water to acid. A friendly pharmacist (not Boots, their always too busy) should be able to advise you how much water you need to achieve a 3.5% solution. Hydrochloric acid for swimming pools come in much larger bottles – far too much. You need to apply tape because this your first time. The screw is the most important part. You don't want to dissolve the metal itself. Once you know how to remove just the rust, on the lesser import parts, then you can tackle removing 'just' the rust on the screw. Then take it out and wipe it with steel wool. After the acid bath, it will be very clean in deed, so immediately apply some oil or grease, to prevent re-rusting. The nut may well be brass or bronze -if it is a good vice. One need not use acid on them, as all they may need is a good clean, in paraffin (kerosene). Likewise, oil or grease immediately. Lastly, be careful of telling your friends and neighbors in the UK that you are doting on your old 'vice' – they may take it the wrong way.Aspro (talk) 00:25, 8 April 2013 (UTC)[reply]
Hydrochloric acid is readily available as a swimming pool chemical. You'd have to determine the concentration to work out how much to dilute it for the desired concentration. — Quondum 23:09, 7 April 2013 (UTC)[reply]
When sold as a swimming pool chemical, hydrochloric acid is often called "Muriatic Acid"...same stuff. SteveBaker (talk) 16:09, 8 April 2013 (UTC)[reply]

Sleep Deprivation

So, I believe I have the new record for number of consecutive days without sleep. I was awake for a period of eighteen consecutive days starting from approximately 9am, June 1st, 2012 and ending at approximately, 4am, Central Standard Time June 19th, 2012. During the course of this ordeal, I was subjected to a small variety of medication prescribed and given to me by a psychiatric in San Antonio, Texas. The reason for my involvement with the psychiatric firm was to treat a simple sleep loss issue and not obviously to be writing this email.

I emailing the "Help" desk with the Wikipedia to inquire how to legitimize my claim and update the Wikipedia. — Preceding unsigned comment added by 67.11.190.179 (talk) 22:39, 7 April 2013 (UTC)[reply]

See Randy Gardner (record holder). Guinness World Records no longer maintain this as a live record. The unofficial record, according to our article, is 18 days, 21 hours, 14 minutes (longer than your 17 days 19 hours, I'm afraid). Tevildo (talk) 22:54, 7 April 2013 (UTC)[reply]
Randy Gardner also set his record without any stimulants, so you'll have to try again, with the proper documentation, and without the pharmaceuticals from the psychiatric in San Antonio, Texas. 202.155.85.18 (talk) 02:00, 8 April 2013 (UTC)[reply]
I doubt it's a good idea to be encouraging this sort of thing. ←Baseball Bugs What's up, Doc? carrots→ 02:52, 8 April 2013 (UTC)[reply]
See also Fatal familial insomnia. ←Baseball Bugs What's up, Doc? carrots→ 00:00, 8 April 2013 (UTC)[reply]
Without documentation in the form of continuous EEG recordings, claims like this are essentially meaningless. Lots of people think they are continuously awake when they are actually taking intermittent short naps. I'm not asserting that that's the case here, just that without evidence there is no way of proving it one way or the other. Looie496 (talk) 00:36, 8 April 2013 (UTC)[reply]
Very possible. Even so, I wonder whether such feats could be possible by unihemispheric slow-wave sleep. That is a phenomenon only known to occur in animals - but a very broad range of animals, which tends to make me wonder if the species that display it are the extreme expression of a possibility that remained in all the transitional forms between. Wnt (talk) 05:10, 8 April 2013 (UTC)[reply]
Giulio Tononi argues that it is possible for the sleep state to be restricted to particular parts of the brain -- for example he interprets sleepwalking as a state where the frontal part of the brain is asleep while the posterior part is awake. So I don't think we can rule out things like that. But I'm not aware of any evidence for anything like unihemispheric sleep in humans. Looie496 (talk) 15:26, 8 April 2013 (UTC)[reply]
"Whether or not the brain is asleep or awake is not an either-or proposition, according to some scientists." Also: "If this partial-sleep hypothesis is correct, some parts of the brain may be asleep while we actually appear to be awake, and vice versa."[9] Bus stop (talk) 15:43, 8 April 2013 (UTC)[reply]
Indeed. However, the parasomniac activities described in many of these cases are hard to qualify as wakefulness, and it is not clear that the person can go indefinitely in such a state. The unihemispheric mechanism seems to offer a way for an organism to maintain what we might agree is "a level of" wakefulness that is consistent for long periods. But if humans can do it at all it is clearly very rare - just plausible enough that, if reported, I could seriously consider that somewhere there could really be someone who ceaselessly chants from his holy book or something for years on end. Wnt (talk) 16:21, 8 April 2013 (UTC)[reply]

April 8

Science or religion?

If scientists believe that they or humans will eventually know everything, isn't that equivalent to a religion? Or even if they believe that they will know more than they currently do that might not always be true, so that is a belief in something without proof. Or is this Q more appropriate for the humanities desk?68.36.148.100 (talk) 00:02, 8 April 2013 (UTC)[reply]

A belief that humans will eventually know everything is indeed a declaration of faith (which is what a religion is), but what makes you think genuine scientists do belive that? I have worked closely with, and supervised, researchers working at Ph.D level. None of them believed that. Have you not heard of the expression "The more I know, the more I know what I don't know" - a favorite among researchers I know. The second part of your question reflects a common misconception about science on your part. Science is not really about asserting facts, it's more about discovering theories that fit known observations, and thus enabling calculations/predictions of results/outcomes for any practical set of conditions. An example is electromagnetics: Over 100 years of electrical engineering has never thrown up a situation where the theory gave a wrong answer. But just what a magnetic field is, or what an electric field is, is something we don't know. Until the theory does throw up a wrong answer to a situation, scientists aren't much concerned about it being merely a theory, and engineers don't care at all - for them it just works. Ratbone 124.182.150.1 (talk) 00:30, 8 April 2013 (UTC)[reply]
I doubt any scientist thinks it is possible to "know everything", regardless, I think you have two major flaws in the premises of your questions: What scientists believe is not what defines science, and similarly 'believing things without evidence' is not what defines religion. Science is based on only a very few fundamental premises, like methodological naturalism, which them selves are not "arbitrarily" chosen, but refined and condensed through hundreds of years of philosophy. Even now those fundamental premises are NOT taken for granted or completely beyond question. If there was any good reason to question or alter those fundamental premises, scientists would do so. Trying to equivocate science to religion is almost only ever done by the religious in an attempt to demote science's privileged epistemological position to 'their' level, which I think is quite funny when you think about it. You'll never hear a scientist accuse a religion of being just another "science". Vespine (talk) 00:41, 8 April 2013 (UTC)[reply]
doesn't that mean that if any scientists have this belief that it is religion to them? I hope for scientists' sake none do. I bet there are a multitude of references of legit scientists who have alluded to these "beliefs" as part of their "work"68.36.148.100 (talk) 00:49, 8 April 2013 (UTC)[reply]
You already asked this question, and we have already answered it. Why don't you go look for those references, since you have faith that they exist? Ratbone 124.182.150.1 (talk) 01:08, 8 April 2013 (UTC)[reply]
why so hostile , isn't this a legit question? Any way I'm too busy watching wrestle mania with my son. And I know those references exist but I don't do reference or bibliographies68.36.148.100 (talk) 01:18, 8 April 2013 (UTC)[reply]
Definitions of "religion" are at wikt:religion, and additional definitions are accessible via http://www.onelook.com/?w=religion&ls=a.
Wavelength (talk) 01:38, 8 April 2013 (UTC)[reply]
Holding a belief, without any evidence, even a dogmatic one, may be a necessary condition of religion, but it's not a sufficient one. Vespine (talk) 01:48, 8 April 2013 (UTC)[reply]
Working scientists usually take for granted a set of basic assumptions that are needed to justify the scientific method: (1) that there is an objective reality shared by all rational observers; (2) that this objective reality is governed by natural laws; (3) that these laws can be discovered by means of systematic observation and experimentation. Philosophy of science seeks a deep understanding of what these underlying assumptions mean and whether they are valid.

The belief that all observers share a common reality is known as realism. It can be contrasted with anti-realism, the belief that there is no valid concept of absolute truth such that things that are true for one observer are true for all observers. The most commonly defended form of anti-realism is idealism, the belief that the mind or consciousness is the most basic essence, and that each mind generates its own reality.[17] In an idealistic world-view, what is true for one mind need not be true for other minds. — Preceding unsigned comment added by 68.36.148.100 (talk) 02:38, 8 April 2013 (UTC)[reply]

What's your question? ←Baseball Bugs What's up, Doc? carrots→ 02:51, 8 April 2013 (UTC)[reply]

I was wondering why scientists do what they do. To make things bigger faster stronger, etc. I came to the conclusion that it is for the betterment of mankind. But obviously there are many reasons. Although ultimately there needs to be that same belief that the future state can be better than the present one. A faith in the natural universe being further understood or revealed for our benefit. 68.36.148.100 (talk) 04:16, 8 April 2013 (UTC)[reply]

If it has worked in the past and you have no good reason to think it will not work in the future, it's not necesarrily an illogical assumption. Especially if the alternative is to sit on your hands in a solipsistic coma. Also, strictly speaking, science doesn't make things better, science is used to discover things about the world. Scientific discoveries can then be applied to make things better, applied science is considered engineering, the fruits of which are technology. Vespine (talk) 04:38, 8 April 2013 (UTC)[reply]
There is a difference between natural science and supernatural science, where science is "knowing". Natural science describes the ordinary functioning of the universe according to its observable laws. Supernatural science can describe divine interventions that alter this normal functioning, or alternate universes or dimensions not accessible to experiment. However, the most important aspect of religion may be in the interpretation of what we do - often the facts are clear, but the moral reaction is anything but agreed upon. Wnt (talk) 04:42, 8 April 2013 (UTC)[reply]
Your assumption that scientists believe that they or humans will eventually know everything is a dubious premise that you have not established, and which looks a lot like a straw man fallacy. What is much less dubious is that scientists believe that through science, scientists or humans will know more than they do currently, which is all that is necessary for a career in science to be a worthwhile endeavor. And that belief is not just a matter of faith. In science, the thing that determines whether or not something is to be believed is scientific evidence, and there is rather trivially evidence that science will cause additional things to become known in the future. Scientists learn new things every day, and there is no evidence that would suggest that tomorrow, that might all stop happening.
The argument against believing certain hypotheses proposed by religions isn't really a matter of thinking that science will eventually have all the answers, so the answers given by religion should be ignored, but rather a matter of considerations like the answers that are given by religion often being contrary to the available evidence, or the answers being given by religion going against Occam's razor by hypothesizing a bunch of stuff which has no predictive value due to the hypothesized stuff being unfalsifiable. Red Act (talk) 05:45, 8 April 2013 (UTC)[reply]
Q1: If scientists believe that they or humans will eventually know everything, isn't that equivalent to a religion?
Bad premise. Scientists not only do not believe that we'll eventually know everything, they know for absolute certainty that they cannot possibly ever know everything.
  1. We have Gödel's incompleteness theorems that says that there are things in mathematics that are true that can neither be proved nor disproved.
  2. The total amount of information about the universe requires something at least as complex as the universe to store it all in. So we can never know every detail.
  3. We have quantum theory that says that we can never know the precise position and momentum of a particle at the same time - and which forces randomness into the universe in fundamental ways. This will always limit our ability to know everything.
  4. Chaos theory which proves that some systems are fundamentally unpredictable: For example: we can never predict the weather more than a few weeks or months in advance.
Science has shown that science has limits.
Q2: Or even if they believe that they will know more than they currently do that might not always be true,
It's obvious that our state of knowledge is always going to be in a state of flux. We are always learning new things - and there is little reason to suppose that we'll ever run out of things like that. But on the other hand, we're continually forgetting things too. It seems certain that one day we'll arrive at a steady state condition where we're forgetting things as fast as we're learning them - but we're nowhere near that point yet.
Q3: so that is a belief in something without proof.
No, I don't see that. Scientists don't claim that we'll eventually know everything - quite the opposite in fact. So your premise is flawed and you can't produce this conclusion from it.
SteveBaker (talk) 16:03, 8 April 2013 (UTC)[reply]
One of the big differences between religious belief and scientific belief is that science is effective. You don't see industrialists gathering groups of theologians to study religious works and from the fruits of that building a better car engine or getting clean water or anything really where one can put down money and bet on which will give a better outcome. Would you say industrialists as engaging in a religious belief? What scientists do is in essence no different. Dmcq (talk) 00:35, 9 April 2013 (UTC)[reply]

Logic, creativity and ignorance

Some claim that logic can explain everything and that when new things are discovered, it is simply ignorance to that logic which has been overcome. I agree but I don't think logic always works because its basically thinking inside the box, I.e. following a set of known principles. Is my thinking correct? — Preceding unsigned comment added by 2.124.100.141 (talk) 00:35, 8 April 2013 (UTC)[reply]

Logic is an extremely broad subject, I think you might be conflating different uses of the term. At its core, formal logic is described as either valid or fallacious, that makes it clear it doesn't have anything to do with inside or outside the box. Fallacious logic will never lead to justified conclusions, it can by chance lead to correct conclusions, but they will not be justified. On the other hand, some discoveries can seem "illogical", like the fact that a photon can have seemingly incompatible properties of waves and particles, outside the box thinking was required to come to terms with that discovery, but that isn't the same meaning of the word logic. In that case, faced with the evidence that was present, outside the box thinking was in fact logical. Vespine (talk) 01:00, 8 April 2013 (UTC)[reply]
Logic doesn't imply that you cannot question your principles. Logic is mainly concerned to deduction from some assumptions to some conclusions. If the assumptions are wrong, everything is wrong. On the other hand, any explanation has to be logical. If you find a contradiction somewhere, it's time to drop some part of your theory.OsmanRF34 (talk) 01:12, 8 April 2013 (UTC)[reply]
I think these are actually very good inquisitive questions, you are starting to scratch the surface of some pretty deep philosophical ideas, about epistemology, metaphysics, etc... If you are genuienly interested, I would strongly recommend something like the free Philosophy for Beginners introduction course from Oxford University. I'm only a novice to philosophy and found these extremely enlightening. Vespine (talk) 01:21, 8 April 2013 (UTC)[reply]
Logic, by itself, cannot explain much. You need to feed it some "axioms". So, for example, no amount of logic will tell you whether the earth is flat or round - you have to feed it some observations (such as the fact that the earth's circular shadow is cast onto the moon during an eclipse - or that ships disappear over the horizon gradually, with their hulls disappearing before their masts). Logic (and more broadly, mathematics) is a tool to help you organize thoughts and data and to extract conclusions and expose weaknesses in "obvious" arguments. Science proceeds with observations and experiments - and logic and mathematics are mostly just tools to help in the process. Logic can be used to produce crazy answers if you give it crazy assumptions. For example, if you start with the assumption that zero is equal to one (or true implies false or something else which isn't true in the real world) - then the rules of logic will happily crunch away on that and produce unending streams of nonsense. This is actually occasionally useful. For example, we know that the three angles of a triangle add up to 180 degrees out here in the real world - but if you deliberately set up a logical system in which that fact is denied - then what emerges is some very interesting non-euclidean geometries that prove useful for all sorts of things. SteveBaker (talk) 16:26, 8 April 2013 (UTC)[reply]
Another way to put this is to contrast deduction with induction. (Forget Sherlock Holmes, he had them backwards.) Induction means (more or less) reasoning to a large conclusion from smaller ones. In other words, I go into the world, I see five birds of the same species, I use them to generalize for the whole species (because I cannot actually look at every member of the species). Deduction means (more or less) reasoning to small conclusions from larger ones. In other words, I know that all birds have feathers, and when asked whether a specific bird (say, a duck) has feathers, I can logically deduce that this is true, a priori. Deduction is generally the realm of formal logic: I know certain things, thus I know other things. But as you can see with my examples given, deduction alone is pretty silly when you are trying to talk about things in the real world and not hypotheticals. How do I know, a priori, that all birds have feathers? Could there be a species out there which does? Deduction alone doesn't let you answer that question. It's why induction became the name of the game for science — go out in the world, see what's there, then work backwards to find what you presume are the "real" rules, and from there you can move tentatively forward with deduction, at least up until you reach the limits of what you know. Induction isn't perfect by itself (see problem of induction — what if the five birds I looked at were, coincidentally, not at all representative?), and neither is deduction (we don't really have a formal problem of deduction but it's basically "how do you know your axioms are correct in anything other than pure philosophy or mathematics?"), but together they are pretty powerful. --Mr.98 (talk) 17:53, 8 April 2013 (UTC)[reply]
Yes, exactly. Formal logic can say:
  • AXIOM: All birds have feathers.
  • AXIOM: A duck is a kind of bird.
  • DEDUCTION: All ducks have feathers.
Which is great - logic told us something that we may not have known at the outset. It's not so obvious that it helped in this case - but for more complex systems where the conclusion is perhaps not so clear, application of formal logic can tell you things you didn't know and wouldn't have guessed.
But: that exact same set of formal logic can go horribly wrong:
  • AXIOM: All birds can fly.
  • AXIOM: An ostrich is a kind of bird.
  • DEDUCTION: All ostriches can fly.
Clearly this is an incorrect deduction - because the first axiom isn't true. This system of formal logic has no way to know that. But that doesn't make formal logic wrong or useless: what it's telling you in this case is NOT that ostriches can fly. It's saying that if all birds can fly - then ostriches can fly...which is a true statement, albeit not a very useful one!
So logic is a tool - and a very handy one - but it's not enough by itself to tell us anything at all about the world. Some poor biologist has to go out there and slog his/her way across the planet, looking at all the birds to see if there are any flightless ones out there - then come back and rewrite that first axiom. Now we have:
  • AXIOM: Some birds can fly.
  • AXIOM: Some birds cannot fly.
  • AXIOM: An ostrich is a kind of bird.
  • DEDUCTION: <nothing>
SteveBaker (talk) 19:04, 8 April 2013 (UTC)[reply]
Can I throw the paradox of the ravens and material implication into the mix? Thanks. Tevildo (talk) 19:48, 8 April 2013 (UTC)[reply]
And the problem of our ability to deal with postulates like "Everybody agrees that a unicorn has one horn" and "Everyone agrees that unicorns do not exist" simulataneously, without being frozen like a robot in a 60s TV show mumbling "Does not compute... does not compute" Gzuckier (talk) 06:22, 9 April 2013 (UTC)[reply]

Rolled up dimensions

I am trying to get my head around the concept of "rolled up" dimensions found in string theory. Higher dimension problems can often be visualised using a two-dimensional world as an analogue and then considering the effects of a third dimension upon it. I am having a failure of imagination here and cannot see how the third dimension could be "rolled up". I have seen Steve Baker's clever analogue of a one-dimensional world in the archives where he rolls up the second dimension into a cylinder around the one-dimensional line people, but I cannot see how this analogy is extended to two-dimensions. Further, a cylinder is three-dimensional, a third dimension was required to enable the second dimension to wrap. In a two-dimension analog would a fourth dimension be required to enable the third dimension to be wrapped? SpinningSpark 11:56, 8 April 2013 (UTC)[reply]

Its simply an analogy because we live in 3-spacial dimensions and related to this is the holographic principle, which I have not studied. Although we generally work with three spacial dimensions, I tend to think of space as one-dimensional though, because I can slice it up into countable discrete volumes with division: simply cut it up into discrete unit volumes such as bricks or tiles and then cut each of these into smaller pieces recursively into infinitesimal volumes. For me, its an interesting conceptual exercise, even if its not all that useful (its utility may be limited to object order algorithms). Of course, with the physics of string theories, one has to consider the dimension of time as well with Einstein's principles and the concept of spacetime. -Modocc (talk) 12:43, 8 April 2013 (UTC)[reply]
It has nothing to do with the holographic principle, which is about information in a volume of space. Space is not 1 dimensional in any meaningful way, even for object order algorithms (because you can't tell what a voxel's neighbors are).
To the OP: The ant on a cylinder, say a garden hose, sees 2 dimensions: one along the cylinder axis and one along the circumference. We're imaging the ant to be a 2D inhabitant of the cylinder surface. If you want the third dimension to be rolled up, our ant has to be already living in 3 dimensional space, but then (as you said) you need to embed this 3D space into a 4D space to visualize it. Our universe is one example of a curved 3D space*, but although you can measure the effects of this curvature, you can't imagine what it looks like unless you can imagine 4D. Similarly, it's pretty easy to predict what a 3D ant would see if the third dimension were rolled up: just read Flatland. --140.180.248.141 (talk) 14:31, 8 April 2013 (UTC)[reply]
*Actually, our universe is very close to having no curvature, but the point is that it would be highly curved if the universe had more mass. --140.180.248.141 (talk) 14:31, 8 April 2013 (UTC)[reply]
Sure a one-dimensional space is meaningful. As I said, a space can be divided iteratively with an algorithm. If I systematically number all the bricks and you select the ith brick, I certainly have enough information to tell you, based on the algorithm used, what its exact neighbors happen to be. Its not necessarily as efficient as linear representations, but its fundamentally sound. As for the holographic principle, its my understanding that it came into play because string theorists discovered that their different dimensional models were related. --Modocc (talk) 14:40, 8 April 2013 (UTC)[reply]
That's not workable. Sure, you can turn a 2D chessboard into a 1D representation by numbering the squares from 1 to 64 - but for an arbitrary point on a 2D plane, you need two numbers...that's fundamental. You can't represent an arbitrary point on a 2D surface with one number - or in 3-space with less than three numbers...it's impossible. The multi-dimensionality of the universe isn't something you can argue away like that. SteveBaker (talk) 15:28, 8 April 2013 (UTC)[reply]
Its simply a matter of utilizing a different representation of a line, plane or volume. I can for instance divide a line into a grid of segments by mapping them to natural numbers, for exampple: {(O,1),(0,.5),(.5,1),(0,-.5),(-.5,-1),(0,1/4),...,(1,2),(1,1.5),(1.5,2),...} For a volume, you can systematically divide the bricks into smaller units likewise with each iteration as well as continue adding unit bricks to the perimeter of the space. Note that each element is uniquely numbered, whether it be a segment, area or volume and each has the exact same dimension as other elements. Also, although points and infinities are not represented, these are not even measurable. -Modocc (talk) 15:55, 8 April 2013 (UTC)[reply]
If you're only interested in labelling every point in nD space, you can just take the decimal representations of each coordinate (x, y, z, w, etc.) and then interleave them (e.g. (0.763, 0.184, 0.952) becomes (0.719685342)). That gives you a unique symbol number for every point in the space. Double sharp (talk) 16:05, 8 April 2013 (UTC)[reply]
It's better than that: you can take a single continuous 1-dimensional curve, and use it to fill up all of space. That's called a space filling curve. This doesn't change the fact that space is 3 dimensional, and that while it might be fun and interesting to represent space with a single real number, it has no physical significance. The 3D nature of space, on the other hand, makes a fundamental difference to the laws of physics. --140.180.248.141 (talk) 16:15, 8 April 2013 (UTC)[reply]
Working with most coordinate systems definitely makes for simpler laws! Although, I tend to distinguish between whatever space is that we measure (and other things as well) and our models or representations of it. -Modocc (talk) 16:35, 8 April 2013 (UTC)[reply]
I cannot see that the 2D ant on the surface of a cylinder works as a 2D analog of the 3D situation. We cannot readily detect that our 3D space is anything other than flat. The ant, on the other hand, will pretty rapidly detect that her second dimension is anything but flat simply by circumnavigating the cylinder. And yes, I have read flatland, and its completely useless for actually understanding anything and certainly doesn't deal with rolled up dimensions. SpinningSpark 15:05, 8 April 2013 (UTC)[reply]
It's not supposed to be a 2D analog - it's a 1D analog. One dimension with a "rolled up" second dimension. The ant in the analogy is a 1D creature - aware only of the direction along the length of the hosepipe. When the "rolled up" dimension around the circumference of the hose is large - then the ant can indeed detect that it's there so he knows he's living in a rather odd 2D world. But the rolled up dimensions envisaged by things like string theory are curved up much more tightly than that...the "diameter" of these extra dimensions would be microscopic...much, MUCH less than the diameter of an atom. The 1D ant can't perceive that he's walking around the circumference of the hose because the diameter is smaller than he can see or feel or anything.
I agree that "Flatland" is a horrible analogy - it's more about the political views (including horrific class bias and appalling misogeny!) of it's author - which are quite painful to read to modern eyes. He's also confused about how his 2D world works, talking about walls, doors and roofs of his 2D houses. A *MUCH* better book about that is "Planiverse" by A.K.Dewdney - I could spend hours looking at the drawings of 2D machines, musical instruments, spacecraft and such. How does a 2D creature eat and defecate without falling in half?! It's exceedingly well thought-out and the story/plot ain't bad either. SteveBaker (talk) 15:40, 8 April 2013 (UTC)[reply]
(What's wrong with class bias and misogyny? It gives you a perspective on a historical society, allowing you to be grateful for modernity.)
To expand on what SteveBaker said, I brought up Flatland because the entire point of a "curled up" dimension is that you don't know about it until you go to the smallest lengths. If we lived in a 3D world where the third dimension was curled up, for example, we wouldn't know about it, and it would look exactly like a 2D world, aka Flatland. --140.180.248.141 (talk) 16:15, 8 April 2013 (UTC)[reply]
Yeah - by all means read Flatland if you're interested in the weird political views of the time - but if you want to get a feel for how a 2D world might be - dump that book in the trash immediately!
If we lived in a universe with a 'curled up' extra dimension, then there are several possibilities:
  • If we had two normal 'flat' dimensions and the radius of curvature of the 3rd dimension was much larger than the size of the visible universe, then we'd be in a world pretty much exactly like the one we live in. Without some very fancy astronomy, we'd never know it. In fact, there is a good case to be made that all three of our spatial dimensions are really like that.
  • If the radius of curvature of a fourth dimension were much smaller than the diameter of an atom (as string theory suggests) then we'd believe that we were living in a 3D world - and it would be exceedingly hard to devise experiments to prove otherwise (which is why we can't prove or disprove string theory right now).
  • If the radius of a fourth dimension was something on a "human scale" - an inch...ten feet...a mile or two...even a few tens of lightyears - then the universe would be an exceedingly weird place. It's fun to try to imagine what that would be like...but it's clear that there are no hidden dimensions that work like that in our universe.
SteveBaker (talk) 16:45, 8 April 2013 (UTC)[reply]
My original analogy (I was going to repeat it - but you dug it up from the archives!) helps me to understand it...but because we have such a hard time wrapping our brains around a universe with four or more spatial dimensions - it's truly necessary to pursue a lower-dimension analog. The "hose-pipe-world" of a 1D creature with a curled up second dimension goes some way to explaining what a curled up 4th dimension might be like. As we imagine the diameter of the hose shrinking down to something less than the creature can perceive, the 1D approximation of reality for that creature is perfect. Our 1D "ant" can only meaningfully move in 1 dimension and the existence of that curled up second dimension is just not noticeable to it.
It's true that to follow that analogy, we need to visualize the plight of our 1D creature as a 3D thing - that's because the concept of "rolling up" entails curving the second dimension into a cylinder - which is a 3D object. Hence we can't imagine a 1D world with a rolled up 2nd dimension by drawing it on a flat 2D plane (like a piece of paper).
That makes visualizing a 2D world with a rolled up 3rd dimension very tough for us...we almost need a 4th dimension in order to see it in our mind's eye. That doesn't invalidate the analogy...it just poses a problem for our imaginations.
I suppose we could try though:
  • Imagine a large room with a very low ceiling...a REALLY large room. The ceiling is so low that we have to crawl around on our bellies to be able to live inside it. This can be considered as something close to a 2D world for us. Let's turn off gravity too (in our mental world, it operates in the 3rd dimension and that confuses this analogy).
  • Now imagine that there are many holes in the ceiling and floors - and that something magical happens. When you move through a hole in the floor - you reappear through a nearby hole in the ceiling...and if you climb up through a hole in the ceiling, you reappear through a nearby hole in the floor. (Just like in that video game "Portal" if you put one portal in the ceiling and the other in the floor just below it).
  • But it's not really "magic". What's happening is that in this world, the 3rd dimension is curled up into a loop and the "circumference" of that 3rd dimensional loop is the same as the height of the ceiling...just a couple of feet.
  • For you, the act of travelling through one of these holes is *almost* like nothing happened...you moved just a foot or two in the third dimension and came back more or less where you started - so moving in the 3rd dimension doesn't do much to your position within the 2D room.
  • Now, imagine the ceiling getting lower and lower...the effect of falling through a hole (in the 3rd dimension) has less and less effect on you.
  • When the 3rd dimension is microscopically thin (and the ceiling is similarly insanely low) - then moving through that 3rd dimension is just like nothing happened. Imagine the ceiling is much lower than the size of an atom...that's how big these hypothetical "curled up" dimensions must be if they really exist.
  • Now, instead of holes, the ceiling and floor are completely permeable...you can pass through them at will at any point - looping around the 3d dimension and arriving back almost exactly where you started.
  • Beings in this world would not know that there was a 3rd dimension...they would assume that the ceiling had zero height (because it's too small for them to perceive). They wouldn't be able to see any distance through the 3rd dimension because photons would also loop around it and arrive back at the same place.
Now stretch the analogy to a 3D world - with a coiled up 4th spatial dimension. We can freely move in the 4th dimension - but the motion is so slight that we can't measure it - light might travel around this 4th direction - but the curvature is so tight that you can't tell. It's a small step from that to imagine many coiled up extra dimensions...dozens of them...enough to make string theory work in fact.
It's a bit of a mental stretch - but it can be done.
SteveBaker (talk) 15:28, 8 April 2013 (UTC)[reply]
The analogy makes sense, but there's something that doesn't quite sit right. According to this analogy, if the three dimensions are equivalent, I should be able to shine a flashlight (or at the atomic scale, emit a photon) at an 80 degree angle toward the ceiling, and it should spend most of its time going up and around. So why don't we see random photons from a source moving more slowly than c? Wnt (talk) 16:10, 8 April 2013 (UTC)[reply]
I wonder that too. It's an interesting question. Perhaps the problem is that you can't arrange to launch a photon in that direction to start with...(naively - the flashlight won't "fit" into the extra dimension - so it has to point nearly parallel to the unwrapped dimensions) - so the error in speed is too small to measure. Are there processes which emit photons in completely random directions? I don't think so - but I might easily be wrong. SteveBaker (talk) 16:31, 8 April 2013 (UTC)[reply]
Since it is an analogy, I wouldn't worry too much about whether or not a flashlight or other stuff fits into it. -Modocc (talk) 17:28, 8 April 2013 (UTC)[reply]
Thanks Steve, that really does make a lot of sense to me. By the way, I realised your hospipe analogy was a 1D analog (as I think my original question makes clear) but I was replying to IP 140 who seems to be offering a 2D hospipe analogy which I still think doesn't work. By by the way, I have read Planiverse as well and agree it is much more interesting from an engineering perpspective, if not a literary one. SpinningSpark 17:15, 8 April 2013 (UTC)[reply]
I was offering the exact same analogy as Steve. The ant lives on the 2D surface of a garden hose, but one of the dimensions is contracted, so it's as if the dimension didn't exist. To the ant, the world seems 1D. I believe that's exactly what Steve is saying as well. --140.180.248.141 (talk) 19:06, 8 April 2013 (UTC)[reply]
Well in that case it wasn't an answer to my question - I already understood the 1D case as I thought I made clear - so its not surprising I was confused. SpinningSpark 21:22, 8 April 2013 (UTC)[reply]
I spent exactly 1 sentence going over Steve's analogy. My point was that in Steve's analogy, the ant's "universe" is 2D, but has one rolled-up dimension. The rest of my post is about why you can't visualize a "universe" that's 3D and has 1 rolled-up dimension: because in order to visualize the curvature of a 3D universe, you need 4 dimensions, just like how you needed 3 dimensions to visualize the ant's 2D universe. Then I explained what an inhabitant of such a 3D universe with 1 rolled-up dimension would see. They would see exactly what the Flatland inhabitants would see, because they'd think their universe was 2 dimensional. Steve said essentially the same thing later on, but in more detail. --140.180.248.141 (talk) 22:14, 8 April 2013 (UTC)[reply]
Well actually I can visualise it after Steve's explanation. SpinningSpark 23:18, 8 April 2013 (UTC)[reply]

You could also twist them and then roll them up so that you get the topology of a moebius strip, see here for a funny article about that. Count Iblis (talk) 23:41, 8 April 2013 (UTC)[reply]

Citing retractions

Hi,

Probably the wrong desk, but I figure someone here is most likely to know the answer. My question is, what's the proper way to deal with a retracted article when writing a publication. I'm writing a thesis about a topic which has previously been studied in an article which has since been retracted. So I need to mention this article. The options I can think of are:

  • Mention the article but don't put it in the refs.
  • Mention it and reference it normally.
  • Mention it and reference the retraction (what's the format for that!?)
  • Don't mention it all, despite them basically doing the same experiments as me.

I haven't found any official guidelines, does anyone know what the best approach would be?

Cheers,

Aaadddaaammm (talk) 14:13, 8 April 2013 (UTC)[reply]

I would reference the article as usual: however, I would mention the fact that the article has since been retracted, either in the body of the work, or in a footnote. If you have the date of the retraction, I think I'd put that at the end of the reference, something like this: "Article X, published in the Journal of Y dated xxx, retracted in the Journal of Y dated zz". --TammyMoet (talk) 14:25, 8 April 2013 (UTC)[reply]
I don't know about "official guidelines" but it seems to me that when a paper is retracted, it is precisely because the authors or publisher no longer believe it should be referenced. Our Retraction article states that: "In science, a retraction of a published scientific article indicates that the original article should not have been published and that its data and conclusions should not be used as part of the foundation for future research. ". For me, "should not have been published" means that you should not reference it...it has (effectively) been "unpublished" and should be considered not to exist - and "should not be used as part of the foundation for future research" is a strong indication that you shouldn't be basing anything on it - and therefore should have no cause to reference it. By referencing a retracted article, you're risking people asking whether your work was in some way based upon it - which would be a huge negative for your work. As our article further notes, if the problem with the paper were not extremely serious, a "correction" would have been published...even if the author says "we messed up, this is all wrong", that would typically result in a correction rather than a retraction. Retractions are generally reserved for serious issues like plagiarism or downright intellectual fraud...and such papers really shouldn't be referenced. SteveBaker (talk) 14:51, 8 April 2013 (UTC)[reply]
My view is that a retracted article should be treated as a primary source. You can cite it to support the fact that it exists and contains certain statements, but you can't cite it as a reliable source. For example in our article on the Schön scandal it would be legitimate to cite his retracted papers when explaining the claims that he made. (As a matter of fact, though, only the retraction notices are cited in our article.) Looie496 (talk) 15:14, 8 April 2013 (UTC)[reply]
Actually I misread the question to be about Wikipedia. For a thesis, I would simply cite both the article and the retraction notice. Looie496 (talk) 15:16, 8 April 2013 (UTC)[reply]
Steve Baker has a point, but I don't agree with his approach for all circumstances. You need to consider the possiblity that your reader may discover for himself the article, but not the retraction. Retractions sometimes come long after an article was published. Why was it retracted? Did they make a simple obvious error that got thru somehow? Or was it a very difficult to spot error that a competent work in the field of study would most likey make as well? Or was it retracted because the author(s) faked their data or something? Was it retracted in error - such things can happen? Such things are difficult to spot. You don't want a reader to rely on the retracted article and junk your work, for want of a sentence or two.
How you should cite it depends on what significance it has to your work.
If your thesis agrees with the article, I suggest you cite it normally and include it in the list of references, and include a discussion stating that it was retracted and explain why you believe the retraction should not have occurred. And cite and list the retraction (which I assume was in the form of a letter to the journal in which it was published).
If the article was retracted due to faked or incorrectly recorded data, and there are lots of other directly relevant references which support that the article was wrong, then I think Steve's approach is valid.
As with any writing, whatever the logic or rules may be, there is one fundamental rule: write for your reader. If it happens that whatever your thesis discusses has not had much prior work, and so the reader is certain to discover the article if he does some kind of a search, then you must cite it and list it in your References section, and you must make it clear in your text body it has been retracted (by who? the journal editors? or the author(s)?) and cite the retraction, and list that in your references section. In such a case, you ought to give your own view on the retraction, and summarise where the author(s) went wrong, in a footnote.
Retractions are often just letters to journal editors, but may be short articles. Either way, it makes no difference, cite and list it in the same way as any other reference.
Ratbone 60.230.248.167 (talk) 15:29, 8 April 2013 (UTC)[reply]
I agree that you also have a point! This isn't simple. If you replace the word "retraction" with "correction" - then I absolutely agree with you. But retractions are supposed to be for such egregious problems that the work is beyond redemption. If (as our retraction article suggests) papers are mainly retracted because of:
  • Plagiarism - then you should obviously be able to reference the original article from which the material was plagiarized.
  • Duplicate/concurrent publishing (self-plagiarism) - then it's the same deal, just reference the original work.
  • Serious errors amounting to scientific misconduct. Well, that's the tough one. Did the serious errors result in a paper that's totally devoid of value? You'd think so, or else a simple "correction" would have been released. The tarnished reputations associated with a retraction are sufficiently great that you'd imagine people would fall over themselves to issue a correction notice rather than retracting the entire work.
So I think the only reason to mention a retracted paper at all would be in the context of "Don't read this - it was retracted"...in which case, I'd reference the retraction notice rather than the paper itself. Remember, people often judge the worth of a paper by the number of people who referenced it - and you want to boost the notoriety of the retraction, not lead people to offer undue respect to the original paper.
The tricky corner case (and this case might fall into that category) would be when you re-did an experiment from a retracted paper - but did it "right" this time. You'd certainly want to avoid someone saying "Oh - you did it *that* way? Didn't so-and-so get their paper retracted for doing that?"...so perhaps there is a case then for explaining how come your version is acceptable. But that leads me back to wondering why a simple experimental, numerical or statistical slip up of a kind that you could fix, would result in a retraction rather than a correction of the original paper.
SteveBaker (talk) 17:01, 8 April 2013 (UTC)[reply]
This is gut feeling: I don't think anyone has the right or ability to dictate good practice on this issue. I am inclined to discard the argument about inflating statistics of a retracted paper out of hand. It doesn't matter what its statistics are. That said, it would depend on the citation. If you're writing that "a widely-disseminated finding that Foo1 strongly represses Bar7 was subsequently retracted", the retraction should be enough. If you're writing that "despite the retraction of Joe Bloggs' group's work for an unrelated misconduct issue, their technician was not implicated in the matter, and we were able to replicate his high-quality immunoprecipitation data for Foo1 with an aliquot the group provided us and have subsequently confirmed the identity of the antigen with a commercial antibody", then cite both paper and retraction at the appropriate places in the sentence. Wnt (talk) 19:33, 8 April 2013 (UTC)[reply]
I If a part of the article was not up to the desirable standard, then the journal, the institution or the author would issue a partial retraction. For smaller stuff, an errata or a correction would suffice. For bigger stuff, like being marred by fraud, a total retraction. That already means that the data is not reliable, so don't use it. As [[10]] point out, citing a retracted article without knowing about it can be embarrassing. But what you are doing is citing it although you know it was retracted. That's bad practice. Don't worry about anyone finding the article anyway, since it won't be present in electronic resources. And don't worry about re-using an idea from a retracted article, since it is not copyrighted. OsmanRF34 (talk) 20:00, 8 April 2013 (UTC)[reply]
I'm concerned about your last sentence. As far as I know, copyright applies to retracted articles. I've never seen anything in copyright law that says otherwise. SteveBaker (talk) 20:42, 8 April 2013 (UTC)[reply]
The 'it' in my last sentence refers to 'idea' not to 'retracted article.' I know that the retracted article gets its copyright at the time of writing. However, no matter what flaws it had, I think the OP saw something of value in the retracted article, but it's not sure how to use it. I just meant that ideas as such are not copyrighted. If someone had a good idea and used a novel procedure, but didn't know how to implement it and massaged the data to make it 'work', then the paper will be retracted, but you still can use the same procedure in the right way. OsmanRF34 (talk) 21:27, 8 April 2013 (UTC)[reply]
It's not apparent from his question whether the OP's thesis will be about pure research or applied research. Its also not apparent whether the retracted article was published in a peer-reviewed journal having a board of editors and high repute, or a commercial journal put out by a magazine publishing company (some are peer reviewed and/or subject to subject matter expert editorial reveiew, or even a company house journal. House journals have ranged from mere carefully written advertising through to publications of very high repute, such as Bell System Technical Journal and Australian Telecommunications Research.
For research papers printed in peer reviewed journals of high repute, the probability of human error causing invalid data or invalid conclusions is low. So retractions are unusual and for serious reasons, such as faked data or other fraud. So you should carefully consider whether to ignore or cite a retracted article for the reason that Steve Baker gave.
However, plenty of articles published in commercial journals and occaisonally house journals have relavence to thesis and peer reviewed jornal articles. In commercial journals, human error can be significant, and retractions due to human error and not a deliberate attempt to defraud are relatively common, as are debates and opposing views in the form of Letters To The Editor. In such cases it is often of great value to cite both the article and the retraction, and discuss the error. Often, the error is such that another worker, expert in the field, could also make the same error and in some cases, most likely would.
You just can't be serious if you suggest that any search of the literature on a subject will always bring a reader's attention to retractions. — Preceding unsigned comment added by 124.178.45.169 (talk) 03:27, 9 April 2013 (UTC)[reply]

Hydrochloric Acid

How does the hydronium ion (H
3
O+
) explain the acidity of hydrochloric acid (HCl). Just to help explain what I am thinking: is there something to do with a Bronsted acid? Are Bronsted acids applicable to hydroxide (-OH) reactions?Curb Chain (talk) 15:39, 8 April 2013 (UTC)[reply]

H
3
O+
ion is a proton donor and therefore a Bronsted acid. OH
ion is a proton acceptor and therefore a Bronsted base. Ruslik_Zero 19:30, 8 April 2013 (UTC)[reply]
Acidity can be explained many ways. There are at least three prominent theories of acid-base chemistry, so you need to consider which theory you are working within before answering the question of "How does..." something about acids and bases work. It entirely depends on which model you're working within. None is wrong, but they are all different, and have their different uses. Acid–base reaction#Common acid–base theories explains all three. Incidentally, HCl is an acid according to all three theories.
The Arrhenius theory explains acids and bases based on the ability of a substance to produce certain ions in water solution. So, if something is added to water and produces hydronium ions, it is an acid, while if something is added to water and produces hydroxide ions it is a base. The basic definition of pH comes from Arrhenius theory, as it is based on ion concentrations in water solutions.
The Brønsted–Lowry theory explains acids and bases in relation to each other rather than to water, removing the restrictions of having water present to explain acids and bases. According to Brønsted–Lowry, an acid and a base are defined based on their place in a "model" reaction:
HA + B ⇌ A- + HB+
whereby HA is an acid, and B is a base. On the product side, A- and HB+ are called conjugates of the reactants, thus A- is the conjugate base of the acid HA, and visa-versa. Conjugate acid-base pairs explain Buffer solution behavior, for example. In the case of the reaction:
HCl + H2O ⇌ Cl- + H3O+
HCl is behaving as the acid, while H3O+ is the conjugate acid of the H2O.
Lewis theory explains acids and bases based on the formation of new covalent bonds. The Lewis acid-base reaction is based on electrons which migrate from species with excess electrons (usually in the form of unbonded pairs of valence electrons, or sometimes as pi bonds) towards a species that is electron deficient. The thing with the excess electrons is called the "lewis base", while the thing that is electron deficient is the "lewis acid". In this case the newly-created bond is between the hydrogen atom from the HCl molecule and the oxygen atom in the water molecule, that hydrogen atom being the lewis acid (being bonded to the more electronegative chlorine makes the hydrogen electron deficient) while the oxygen atom has excess electrons in the form of unbonded "lone" pairs, the new O-H bond that turned water into hydronium being the focus of the Lewis theory definition of acids and bases. I hope all of this helps. --Jayron32 19:54, 8 April 2013 (UTC)[reply]
So equilibrium does not exist in Arrhenius or Lewis Theory? Why isn't the relationship between metals such as NaOH (or other hydroxides such as KOH) and NaCl the same?
How is this different from reduction and oxidation reactions?Curb Chain (talk) 03:50, 9 April 2013 (UTC)[reply]
Who said equilibrium concepts didn't exist in Arrhenius or Lewis? I certainly didn't. Also, I'm not sure I follow your question regarding metals and hydroxides is. Could you restate it or elaborate what you're asking about? Also, it is quite different from redox: redox reactions are those which change the Oxidation state of an element. Acid base reactions don't change the oxidation state of anything (if you follow the formal rules for assigning oxidation number to atoms in any acid-base reaction, you'll find that none of them involve changes in oxidation state). But if you could restate your first two questions in detail: what exactly do you not understand or what do you need answered, that'd be great, because I can't follow what you're asking here. --Jayron32 04:21, 9 April 2013 (UTC)[reply]

AIDS in non-human animals

disease like AIDS will affect the other animals as they sexually related to more than one opposite partner? — Preceding unsigned comment added by Titunsam (talkcontribs) 17:19, 8 April 2013 (UTC)[reply]

As our HIV/AIDS article explains, HIV originated in chimpanzees, and similar diseases have been found in other primates. So yes, it is possible. Looie496 (talk) 18:02, 8 April 2013 (UTC)[reply]
Then there's feline AIDS. StuRat (talk) 03:05, 9 April 2013 (UTC)[reply]

What's the study of historical people's illnesses called?

You know like Charles Darwin or George III? Given the medicine at the time didn't know as much as today. Surely it must have a greco-latin type name ending in logy? Barney the barney barney (talk) 19:15, 8 April 2013 (UTC)[reply]

Paleopathology? OsmanRF34 (talk) 19:31, 8 April 2013 (UTC)[reply]
Retrospective diagnosis, rather. Or maybe historical clinicopathology. Deor (talk) 21:34, 8 April 2013 (UTC)[reply]

MBTs Armor

Does Challenger 2 and M1 Abrams have composite armor at their hull sides or it is only at the front of the hull ? — Preceding unsigned comment added by Tank Designer (talkcontribs) 19:55, 8 April 2013 (UTC)[reply]

Don't know about the Challenger, but our Abrams has composite armor all around. 24.23.196.85 (talk) 05:56, 9 April 2013 (UTC)[reply]

not to be annoying, but had more question... (concerning hydrocarbons)

collapsing copy-paste of archived section without wiki-formatting. Looie496 (talk) 20:23, 8 April 2013 (UTC)[reply]
The following discussion has been closed. Please do not modify it.

There are oil eating microbes, and hydrocarbon seep communities, and studies that show that a large percentage of hydrocarbons on the surface are fossil derived. I am casually investigating the possibility that we are negatively affecting our organic fertility by over-harvesting fossil fuels. The "hypothesis" is that fossil fuels, while in the ground, are part of the "microbial food chain", which is in turn responsible for such aspects as soil fertility. The proposed issue is that by oil pumping, coal mining, and gas pumping (lately "fracking") are causing a loss in the ecological capabilities of harvested regions. So, fossil fuel harvesting might be contributing to desertification. As there is less food for life, there is less life. — Preceding unsigned comment added by 134.29.38.118 (talk) 20:14, 1 April 2013 (UTC)

An interesting proposition, but your entire post amounts to a statement. This Reference Desk is intended for asking questions to which responders can give (preferably referenced) factual answers: did you have a question? If so, please ask it explicitly. Conversely, this Reference Desk is not intended for hosting debates and discussions, or soliciting opinions, so if your post was an implied invitation for either of these, it is inappropriate here. {The poster formerly known as 87.81.230.195} 90.197.66.100 (talk) 21:23, 1 April 2013 (UTC)

I think the implicit question is reasonably clear, but I can't see how that could work out. Soil fertility is determined by the top few feet of soil (often much less), but hydrocarbons are found much farther down. Also some of the most fertile soils are found hundreds of miles from any known hydrocarbon deposits. Looie496 (talk) 21:46, 1 April 2013 (UTC)

Agreed. Also, fracking is a problem precisely because it contaminates the groundwater with hydrocarbons, which, under this theory, would be a good thing. StuRat (talk) 22:45, 1 April 2013 (UTC)

...assuming that the other chemicals associated with fracking are OK. Of course, having seen what happens to plants when they are exposed to crude oil, I have my doubts about this whole concept. --Guy Macon (talk) 00:09, 2 April 2013 (UTC)

In response, pollution is unnatural. Natural hydrogeological blah blah would run the hydrocarbons through bacteria, and each step would take them through some modification very slowly, but very steadily. Pollution just puts refined and broken hydrocarbons where they weren't before. There certainly are bacteria all the way up and down, we've got a lot of articles on it over the last decade or so. There are also hydrocarbon seep communities, which are unique, but then food for passing life, and what I'm talking about is basically hydrocarbon seep, but rather than through a hole or fault, just slowly via osmosis. — Preceding unsigned comment added by 66.188.222.186 (talk) 01:52, 2 April 2013 (UTC)

It takes a very special class of organisms to use hydrocarbons as their main source of food; most other organisms (including all multicellular plants and animals) not only cannot use hydrocarbons in this way, but are positively harmed by them, either directly (acute toxicity, sunlight deprivation for aquatic plants, clogging of gills in fish and of feathers in birds, etc.) or indirectly (increased BOD caused by proliferating extremophile bacteria, possible infection by same, etc.) So, in other words, an oil spill is a VERY bad thing -- and in fact, for some of the same reasons that phosphate runoff is a bad thing (which would have been a "good thing" by the OP's reasoning, too). 24.23.196.85 (talk) 01:02, 3 April 2013 (UTC)

start unanswered.... Ok, so it does indeed take a very special class of organisms to use hydrocarbons. The place we originally found them was in the reserves or in hydrocarbon seeps. I'm ballparking. When there has been an oil spill, probably (I recall) microbes then did also proliferate in these locals, and utilize these hydrocarbons. modern science has made strains that are more rapid in their use of the hydrocarbons. rewinding, and commenting also on the soil fertility. it's true, we measure fertility in the first few feet of soil, but the hydro-logical cycle of the subterranean environment means that much more of the earth has an impact on the small region which is this few feet. If there is a water table 60 feet below the surface (Depending on other geological formations) it will have an impact on the soils conditions. I am suggesting that likewise, having gas shale (such as in minnesota) 60 feet below the surface would also have an effect, and here's why. Naturally occurring within the shale are hydrocarbon "eating" microbes, which then convert the hydrocarbons into other forms of organic matter. Through osmosis, and other hydro-logical activity, some of the contents of the shale would be released towards the surface. Although this would occur very slowly, and very little at any given time, it would be like the growth of any other permanent, fixed organic community, and very continuous. My QUESTION is mostly is this accepted as common understanding, and why or why not, and what science supports our common understanding? btw, I am the original poster... thanks! — Preceding unsigned comment added by 66.188.222.186 (talk) 20:03, 8 April 2013 (UTC)[reply]

Fertility is mainly determined by levels of nitrogen, phosphorus, and potassium, along with proper pH and absence of plant toxins; see NPK rating. None of those are present at significant levels in oil or natural gas. There is lots of carbon, but plants get their carbon from atmospheric CO2, not from the soil. Looie496 (talk) 20:33, 8 April 2013 (UTC)[reply]
To answer your question: No, this is not the common scientific understanding. Your ideas are vague and hard to understand, so I cannot say exactly where you go astray of the common consensus. Though bacteria that metabolize hydrocarbons are fascinating (and currently highly studied), there is no "textbook" support for the notion that they make meaningful contributions to the biogeochemistry of terrestrial ecosystems. See also soil fertility and soil biology. As the latter article points out, this is still a very active field of research, and there are certainly things we still don't understand about the ecosystem ecology of the deep earth. SemanticMantis (talk) 02:08, 9 April 2013 (UTC)[reply]
Also note that oil, gas, and coal reserves are millions of years old, so, if they were being eaten at a significant rate, they would be gone by now. The conclusion, they can't possibly be eaten by microbes at a significant rate, at least while sequestered underground. Once on the surface, where sunlight and other forces can assist in the breakdown, then the bacteria can go wild on it. StuRat (talk) 04:57, 9 April 2013 (UTC)[reply]

Orange dwarf stars left main sequence yet

Did any orange dwarf stars left their main sequence stage yet? I thought 0.8 solar mass stars might have left main sequence. Is 0.8 sun like star or is it more like a orange dwarf star? I am confused a little bit. Because stellar classification said orange main sequence stars are between 0.6 and 0.9 solar mass, orange dwarf detailed article said it is 0.5 to 0.8 solar mass. When orange main sequence stars leave main sequence do they go to a red giant? If it goes to a red giant, will it be s smaller sized red giant, or about the similar orb as if sun might go through. When red dwarf leave its main sequence will it become a red giant? Or we don't know what will happen to these red dwarfs when it leaves its main sequence, or more likely they will shrink straight to white dwarf. I am not sure what is Subdwarf B and A is for. If that what red dwarf or orange dwarf might end its life?--69.226.42.134 (talk) 23:27, 8 April 2013 (UTC)[reply]

Red dwarf says:
"According to computer simulations, the minimum mass a red dwarf must have in order to become a red giant is 0.25 solar masses; lesser massive objects, as they age, increase their surface temperatures and luminosities becoming blue dwarfs and from that finally become white dwarfs."
We only have computer simulations, since it will be 10s of billions of years before the first red dwarfs start to die.
An Orange Dwarf is a main sequence star - see K-type main-sequence star. The mass ranges are estimates, so different sources will give slightly different figures (although we should try and be consistent between Wikipedia articles...), but they are describing the same stars by a different name. That article says they remain on the main-sequence for 15-30 billion years. The universe is about 14 billion years old, so presumably none of them have left the main-sequence yet. When they do, I think they will become red giants, although our article doesn't say so (if yellow dwarfs do and large red dwarfs do, then orange dwarfs that are somewhere inbetween presumably will as well). --Tango (talk) 11:15, 9 April 2013 (UTC)[reply]

Evolution of higher mass stars

I am confused about the white dwarf, neutron star and black hole stuff. I thought stars between 1.5 and 3 solar mass suppose to end to a neutron star. When a larger white main sequence star leave main sequence does it become a bright giant or only it becomes a red giant. I thought white main sequence stars (2 or 3 solar mass) will eventually become a red supergiant, or before it hits supergiant it has to go through red giant first? I always learn when large blue stars (between 3 to 8 solar mass) runs out of hydrogen it suppose to become a red supergiant. I wonder how can a blue main sequence stars (3 to 8 solar mass) become just a red giant? I thought they will eventually hit a red supergiant. i am not sure how will they end up with a white dwarf. Or before they reach neutron stars or black hole they will have to go through white dwarf first. I always learned that white and blue main sequence stars end life in neutron star or black hole. --69.226.42.134 (talk) 23:38, 8 April 2013 (UTC)[reply]

Neutron stars are that size but they are the small remnants of massive stars after they supernova. Stars which start out that size end up as white dwarfs. At least as far as I can tell from our articles which could be more clear about mass ranges. Rmhermen (talk) 03:20, 9 April 2013 (UTC)[reply]

April 9

Colour of water

Hi, there was a question once at Colour of water about why the water in the pool in the image at the right looks equally blue regardless of whether one is looking through a few centimetres (at the edge of the pool) or several feet (in the centre of the pool). "Common sense" would suggest that the more water you looked through, the bluer it would be. AFAIK, no satisfactory answer was ever provided. Does anyone have an explanation? 86.171.43.156 (talk)

Water is very faintly blue (or blue-green, in the case of sea water). So, a small amount of it will appear clear, unless blue dye has been added or something blue is behind it, like the bottom of the pool painted blue. In your example, the water in the bucket certainly doesn't look as blue. StuRat (talk) 03:00, 9 April 2013 (UTC)[reply]
When water is deep enough, it appears black. Plasmic Physics (talk) 03:19, 9 April 2013 (UTC)[reply]
I don't see how any of that answers the question. Why does the water in the bucket look much less blue than the same thickness of water at the side of the pool? (We take it on trust that the pool is tiled throughout with the same white tiles as are visible around the edge.) Why does the deepness of the blue not reduce dramatically at the sides of the pool where you are looking through an increasingly small distance of water? 86.171.43.156 (talk) 03:35, 9 April 2013 (UTC)[reply]
What do you mean? Stu just answered that in a direct manner, what part of his answer do you not understand? Plasmic Physics (talk) 03:48, 9 April 2013 (UTC)[reply]
I think they're saying that there is a point near the wall where you are only looking through as much water as is in the bucket, yet it looks far more blue. I suspect the answer is simple scattering. The water in the pool is all scattering the blue reflected off the bottom of the pool. Since the bucket is opaque and white, it will look far more clear. Vespine (talk) 04:05, 9 April 2013 (UTC)[reply]
(ec) I think I understand. He was assuming that the light 6 inches below the waterline shines straight in 6 inches, hits the edge of the pool, then reflects right back out of the pool, just as it does in the bucket. However, most of the light that hits 6 inches below the waterline is reflected downwards to the bottom of the pool, while other light reflects back up from the bottom of the pool, hits that spot 6 inches below the waterline, and back out again. Thus, this light has traveled through a lot more than 1 foot of water total, and had the redder waves absorbed, leaving a distinctly bluish tint.
A horizontal, opaque, white surface, 6 inches below the waterline, should appear white, not blue. StuRat (talk) 04:09, 9 April 2013 (UTC)[reply]

The meltwater pool on top of a glacier does look very blue. [11] --Stone (talk) 08:09, 9 April 2013 (UTC)[reply]

Mus musculus anatomy and behavior, plus trap physics

Some few minutes ago, I heard the snap of the mousetrap in the kitchen, and I went out soon enough that the mouse was still twitching and its tail still moving around. This prompted a few questions in my mind that I've not been able to answer with a Google search:

  • Why does a trapped mouse twitch? Is it something neurological that's comparable to what causes a chicken to move uncontrollably after being beheaded?
  • What does a healthy Mus musculus do with its tail in daily life? Balance? I've only ever seen the species (1) in traps, or (2) running around on my floors, seemingly not using its tail.
  • Why do traps often flip upside down when set off by a mouse? Is the spring so powerful that its momentum forces the trap to go upside down, or is there some other explanation?

Nyttend (talk) 04:42, 9 April 2013 (UTC)[reply]

1) All animals may twitch shortly after death, including humans. The muscle cells remain alive, and have energy, for some time, but only receive random nerve impulses, so move randomly. If you apply an electric current, you can cause the twitching yourself. This is the basis for the fictional idea that you can bring the dead back to life with a lot of electricity, as in Frankenstein.
2) Yes, balance. You'd need some high speed film to see it using it as a counterbalance during quick turns, etc.
3) Yes, the momentum of the spring does it. StuRat (talk) 04:46, 9 April 2013 (UTC)[reply]

Orbital Debris

After reading the "orbital debris" article, and a lot of the references, I just can't quite understand why so much stuff stays there. (1)Surely whenever there's a collision between objects, or even a close pass, they must both end up with a change in velocity that would eventually de-orbit them? (2)Is a "Graveyard orbit" one that is slowly decaying away from Earth? or is it just a stable orbit that's out of the way above the valuable stuff? (3)How big would an orbiting object have to be to actually attract other objects to itself by gravity? I hope someone can help me with this, thanks in advance. 122.108.189.192 (talk) 07:34, 9 April 2013 (UTC)[reply]

If they collide they just get orbits which aren't so circular in general. Reducing the speed of an object that is in a circular orbit puts it into a more elliptical orbit but whatever happens unless it encounters drag from the atmosphere it eventually gets back to where it started (ignoring other gravity effects, pressure from sunlight solar wind, magnetism etc.). For low earth ones that means they will probably decay faster but for higher ones it simply means you have an expanding cloud of debris. The graveyard orbit is just one that's out of the way, I don't think of it as a complete solution myself. The stuff up there is too small and far away from each other for gravitation to have an effect that matters. Dmcq (talk) 08:19, 9 April 2013 (UTC)[reply]
There is no minimum size an object has to be to attract other objects. However, the gravitational attraction of small objects is very easily overcome by other forces. In particular tidal forces prevent the formation of bodies by gravity inside the Roche limit. SpinningSpark 11:19, 9 April 2013 (UTC)[reply]
Over time, collisions will cause objects to either go into low enough orbits that atmospheric drag causes them to re-enter and burn up, or go into high enough orbits that they stop colliding with things. However, collisions in orbit are actually very rare - it would take centuries (at least) for what is up there to clean itself up through collisions.
Graveyard orbits are just high enough orbits to be out of the way. They are potentially still a problem if you want to go through that orbit (to get to geostationary orbit, for example) or if two things in graveyard orbits collide and some of the parts get knocked into orbits that cross low earth orbit (although, the further out you go the more room there is, so such collisions are unlikely).
See Hill sphere for details of stable orbits around satellites. For objects in low earth orbit, the object would have to be denser that any naturally occuring materials for anything to orbit them (it isn't enough for an object to be very massive, it needs to be small too or orbits around it have to be too large to avoid colliding with it). There will still be an effect from gravity even if there isn't a stable orbit possible, but for any realistic satellite it won't be a measurable effect. --Tango (talk) 11:26, 9 April 2013 (UTC)[reply]